Download as pdf or txt
Download as pdf or txt
You are on page 1of 70

Pr

oudl
yPr
esent
s

C
RI
TI
CA
R
L
E
A
R
S
E
O
N
A
I
S
N
G
O
NIN
G
F
orB
ank
in
g,
SS
C,U
PSC
,St
at
ePS
C,R
ai
lwa
y,
Gen
er
al
Comp
et
i-
ti
ona
ndJ
obr
el
at
edex
ams
.

Partners
Critical Ability Set-1
Q.1: Insurance Company X is considering issuing a new policy to cover services
required by elderly people who suffer from diseases that afflict the elderly. Premiums for
the policy must be low enough to attract customers; therefore, Company X is concerned
that the income from the policies would not be sufficient to pay for the claims that would
be made.

Which of the following strategies would be most likely to minimize Company X's losses
on the policies?

A. Attracting middle-aged customers unlikely to submit claims for benefits for many years.
B. Insuring only those individuals who did not suffer any serious diseases as children.
C. Including a greater number of services in the policy than are included in other policies of
lower cost.
D. Insuring only those individuals who were rejected by other companies for similar policies.

Answer Option A

Q.2: French cuisine is highly regarded all over the world. Yet in Paris there are more
American restaurants selling burgers and fries (which many people now class as junk
food) than there are in any other European capital city. Obviously the French are very
fond of junk food, and are not too proud to eat it.

Which of the following, if true, would most weaken the author’s contention?

A. There are also a larger number of Lebanese restaurants in Paris than there are in other
European capital cities.
B. French Cordon Bleu cuisine is very expensive.
C. The number of French tourists eating in New York burger restaurants is very low.
D. There are an unusually large number of American tourists in Paris who eat at burger
joints.

Answer Option D

Q.3: Rye sown in the fall and plowed into the soil in early spring leaves a residue that is
highly effective at controlling broad-leaved weeds, but unfortunately for only about forty-
five days. No major agricultural crop matures from seed in as little as forty-five days.
Synthetic herbicides, on the other hand, although not any longer-lasting, can be
reapplied as the crop grows. Clearly, therefore, for major agricultural crops, plowing rye
into the soil can play no part in effective weed control. The argument is most vulnerable
to the objection that it fails to

A. consider that there might be minor, quick-growing crops that do mature in forty-five days
or less

Website Sources: https://www.edufever.com/ Page 1


Critical Ability Set-1
B. identify any alternative method of weed control that could be used instead of the method
it rejects
C. distinguish among the various kinds of synthetic herbicides
D. allow for the possibility of combining the two weed-control methods it mentions

Answer Option D

Q.4:

Q.5: The Maxilux car company's design for its new luxury model, the Max 100, included
a special design for the tires. That was intended to complement the model's image. The
winning bid for supplying these tires was submitted by Rubco. Analysts concluded that
the bid would only just cover Rubco's costs on the tires, but Rubco executives claim that
winning the bid will actually make a profit for the company.

Which of the following, if true, most strongly justifies the claim made by Rubco's
executives?

A. In any Maxilux model, the spare tire is exactly the same make and model as the tires
that are mounted on the wheels
B. Rubco holds exclusive contracts to supply Maxilux with the tires for a number ofother
models made by Maxilux.
C. The production facilities for the Max 100and thosefor thetires to be supplied by Rubco
are located very near each other.
D. When people who have purchased a carefully designed luxury automobile need to
replace a worn part of it, they almost invariably replace it with a part of exactly the same
make and type.

Answer Option D

Q.6: Certain genetically modified strains of maize produce a powerful natural


insecticide. The insecticide occurs throughout the plant, including its pollen. Maize
pollen is dispersed by the wind and frequently blows onto milkweed plants that grow
near maize fields. Caterpillars of monarch butterflies feed exclusively on milkweed
leaves. When these caterpillars are fed milkweed leaves dusted with pollen from
modified maize plants, they die. Therefore, by using genetically modified maize, farmers
put monarch butterflies at risk.

Which of the following would it be most useful to determine in order to evaluate the
argument?

A. Whether the natural insecticide is as effective against maize-eating insects as


commercial insecticides typically used on maize are

Website Sources: https://www.edufever.com/ Page 2


Critical Ability Set-1
B. Whether the pollen of genetically modified maize contains as much insecticide as other
parts of these plants
C. Whether monarch butterfly caterpillars are actively feeding during the part of the growing
season when maize is releasing pollen
D. Whether insects that feed on genetically modified maize plants are likely to be killed by
insecticide from the plant's pollen

Answer Option C

Q.7: In order to reduce the number of items damaged while in transit to customers,
packaging consultants recommended that the TrueSave mail-order company increase
the amount of packing material so as to fill any empty spaces in its cartons. Accordingly,
TrueSave officials instructed the company's packers to use more packing material than
before, and the packers zealously acted on these instructions and used as much as
they could. Nevertheless, customer reports of damaged items rose somewhat.

Which ofthe following, if true, mosthelps to explain why acting onthe consultants'
recommendation failed to achieve its goal?

A. The change in packing policy ledto an increase in expenditure on packing material and
labor.
B. When packing material is compressed too densely, it loses some of its capacityto absorb
shock.
C. The amount of packing material used in a carton does not significantly influence the
ease with which a customer can unpack the package.
D. Most of the goods that TrueSave shipsare electronic products that are highly vulnerable
to being damaged in transit.

Answer Option B

Q.8: If the county continues to collect residential trash at current levels, landfills will
soon be overflowing and parkland will need to be used in order to create more space.
Charging each household a fee for each pound of trash it puts out for collection will
induce residents to reduce the amount of trash they create; this charge will therefore
protect the remaining county parkland.

Which of the following is an assumption made in drawing the conclusion above?

A. The collection fee will not significantly affect the purchasing power of most residents,
even if their households do not reduce the amount of trash
B. The collection fee will not induce residents to dump their trash in the parklands illegally.
C. The beauty of county parkland is an important issue for most of the county's residents.
D. Landfills outside the county's borders could be used as dumping sites for the county's
trash.

Website Sources: https://www.edufever.com/ Page 3


Critical Ability Set-1
Answer Option C

Q.9: An industry wants to change its policy concerning daily working hours. Currently,
this company requires all employees to arrive at work at 7 a.m. The proposed policy
would permit each employee to decide when to arrive-from as early as 5 a.m. to as late
as 10 a.m.

The adoption of this policy would be most likely to decrease employees’ productivity if
the employees’ job functions required them to

A. work without interruption from other employees


B. consult at least once a day with employees from other companies
C. submit their work for a supervisor’s eventual approval
D. interact frequently with each other throughout the entire workday

Answer Option D

Q.10: Last year the rate of inflation was 1.2 percent, but for the current year it has been
4 percent. We can conclude that inflation is on an upward trend and the rate will be still
higher next year.

Which of the following, if true, most seriously weakens the conclusion above?

A. The inflation figures were computed on the basis of a representative sample of economic
data rather than all of the available data.
B. Last year a dip in oil prices brought inflation temporarily below its recent stable annual
level of 4 percent.
C. lncreases in the pay of some workers are tied to the level of inflation, and at an inflation
rate of 4 percent or above, these pay raises constitute a
D. Government intervention cannot affect the rate of inflation to any significant degree.

Answer Option B

Q.11: Wood smoke contains dangerous toxins that cause changes in human cells.
Because wood smoke presents such a high health risk, legislation is needed to regulate
the use of open-air fires and wood-burning stoves.

Which of the following, if true, provides the most support for the argument above?

A. The amount of dangerous toxins contained inwood smoke is much less than the amount
contained in an equal volume of automobile exhaust.
B. Within the jurisdiction covered by the proposed legislation, most heating and cooking is
done with oil or natural gas.
C. Smoke produced by coal-burning stoves is significantly more toxic than smoke from
wood-burning stoves

Website Sources: https://www.edufever.com/ Page 4


Critical Ability Set-1
D. In valleys where wood is used as the primary heating fuel, the concentration of smoke
results in poor air quality

Q.12: Advertisement: Today's 'customers expect high quality. Every advance in the
quality of manufactured products raises customer expectations. The company that is
satisfied with the current quality of its products will soon find that its customers are not.
At MegaCorp, meeting or exceeding customer expectations is our goal.

Which of the following must be true on the basis of the statements in the advertisement
above?

A. MegaCorp's competitors will succeed in attracting customers only if those competitors


adopt MegaCorp's goal as their own.
B. A company that does not correctly anticipate the expectations of its customers is certain
to fail in advancing the quality of its products.
C. MegaCorp s goal is possible to meet only if continuing advances in product quality are
possible.
D. If a company becomes satisfied with the quality of its products, then the quality of its
products is sure to decline.

Answer Option C

Q.13: While many people think of genetic manipulation of food crops as being aimed at
developing larger and larger , some plant breeders have in fact concentrated on
discovering or producing dwarf varieties, which are roughly half as tall as normal
varieties. Which of the following would, if true, most help to explain the strategy of the
plantbreeders referred to above?

A. Plant varieties used as food by some are used as ornamentals by others.


B. The wholesale prices of a given crop decrease as the supply of it increases
C. Crops once produced exclusively for human consumption are often now used for animal
feed.
D. Short plants are less vulnerable to strong wind and heavy rains

Answer Option D

Q.14: Electricity would revolutionize agriculture according to a prediction of the not-so-


distant future published in 1940. Electrodes would be inserted into the soil, and the
current between them would kill bugs and weeds and make crop plants stronger.

Which of the following, if true, most strongly indicates that the logic of the prediction
above is flawed?

A. In order for farmers to avoid electric shock while working in the fields, the current could
be turned off at such times without diminishing the intended effects
Website Sources: https://www.edufever.com/ Page 5
Critical Ability Set-1
B. If the proposed plan for using electricity were put into practice, farmers would save on
chemicals now being added to the soil.
C. It cannot be taken for granted that the use of electricity is always beneficial.
D. Since weeds are plants, electricity would affect weeds in the same way as it would affect
crop plants.

Answer Option D

Q.15: The Eurasian ruffe, a fish species inadvertently introduced into North America's
Great Lakes in recent years, feeds on the eggs of lake whitefish, a native species, thus
threatening the lakes' natural ecosystem. To help track the ruffe's spread, government
agencies have produced wallet-sized cards about the ruffe. The cards contain pictures.
of the ruffe and explain the danger they pose; the cards also request anglers to report
any ruffe they catch.

Which of the following, if true, would provide most support for the prediction that the
agencies' action will have its intended effect?

A. Ruffe generally feed at night, but most recreational fishing on the Great Lakes is done
during daytime hours.
B. Most people who fish recreationally on the Great Lakes are interested in the
preservation of the lake whitefish because it is a highly prized game fi
C. The ruffe is one of several nonnative species in the Great Lakes whose existence
threatens the survival of lake whitefish populations there.
D. The bait that most people use when fishing for whitefish on the Great Lakesis not
attractive to ruffe.

Answer Option B

Q.16: Hollywood restaurant is replacing some of its standard tables with tall tables and
stool's. The restaurant already fills every available seat duringits operating hours, and
the change in seating arrangements will not result in increase in the restaurant's seating
capacity. Nonetheless, the restaurant's management expects revenue to increase as a
result of the seating change without any concurrent change in menu, prices, or
operating hours.

Which of the following, if true, provides the best reason for the expectation?

A. One of the taller tables takes up less floor space than one of the standard tables
B. Diners seated on stools typically do not linger over dinner as long as diners seated at
standard tables.
C. Since the restaurant will replace only some of its standard tables, it can continue to
accommodate customers who do not care for the taller tables.
D. Few diners are likely to avoid the restaurant because of the new seating arrangement.

Website Sources: https://www.edufever.com/ Page 6


Critical Ability Set-1
Answer Option B

Q.17: A drug that is highly effective in treating many types of infection can, at present
be obtained only from the bark of the ibora, a tree that is quite rare in the wild. It takes
the bark of 5,000 trees to make one kilogram of the drug. It follows, therefore, that
continued production of the drug must inevitably lead to the ibora's extinction. Which of
the following, if true, most seriously weakens the argument above?

A. The drug made from ibora bark is dispensed to doctors from a central authority.
B. The leaves of the ibora are used in a number of medical products.
C. The ibora can he propagated from cuttings and grown under cultivation.
D. The ibora generally grows in largely inaccessible places.

Answer Option C

Q.18: Parland's alligator population has been declining in recent years, primarily
because of hunting. Alligators prey heavily on a species of fresh water fish that is highly
valued as food by Parlanders, who had hoped that the decline in the alligator population
would lead to an increase in the numbers of these fish available for human
consumption. Yet the population of this fish species has also declined, even though the
annual number caught for human consumption has not increased.

Which of the following, if true, most helps to explain the decline in the population of the
fish species?

A. The decline inthe alligator population has meant that fishers can work in some parts of
lakes and rivers that were formerly too dangerous.
B. Over the last few years, Parland's commercial fishing enterprises have increased the
number of fishing boats they use.
C. The main predator of these fish is another species of fish on which alligators also prey.
D. Many Parlanders who hunt alligators do so because of the high market price of alligator
skins, not because of the threat alligators pose to the fish population.

Answer Option C

Q.19: Physician: The hormone melatonin has shown promise as a medication for sleep
disorders when taken in synthesized form. Because the long-term side effects of
synthetic melatonin are unknown, however, I cannot recommend its use at this time.
Patient: Your position is inconsistent with your usual practice. You prescribe many
medications that you know have serious side effects, so concern about side effects
cannot be the real reason you will not prescribe melatonin.

The patient's argument is flawed because it fails to consider that

Website Sources: https://www.edufever.com/ Page 7


Critical Ability Set-1
A. The side effects of synthetic melatonin might be different from those of naturally
produced melatonin
B. Sleep disorders, if left untreated, might lead to serious medical complications
C. The side effects of a medication can take some time to manifest themselves
D. Known risks can be weighed against known benefits, but unknown risks cannot

Answer Option D

Q.20: Traverton's city council wants to minimize the city's average yearly expenditures
on its traffic signal lights and so is considering replacing the incandescent bulbs
currently in use with arrays of light-emitting diodes (LEDs) as the in candescent bulbs
burn out. Compared to incandescent bulbs, LED arrays consume significantly less
energy and cost no more to purchase. Moreover, the costs associated with the
conversion of existing fixtures so as to accept LED arrays would be minimal.

Which of the following would it be most useful to know in determining whether switching
to LED arrays would be likely to help minimize Traverton's yearly maintenance costs?

A. Whether the expected service life of LED arrays is at least as long as that ofthe currently
used incandescent bulbs
B. Whether anycities have switched from incandescent lights in theirtraffic signalsto lighting
elements other than LED arrays
C. Whether anycities have switched from incandescent lights in theirtraffic signalsto lighting
elements other than LED arrays
D. Whether Traverton's city council plansto increase the numberof traffic signal lights in
Traverton

Answer Option A

Q.21: In an attempt to abate the destructive decline in Micro Chip's revenue brought
about by shrinking demand that is accompanying an economic recession, Micro Chip is
offering customers a 50% discount for the next three months on all purchases fully paid
for within 15 days.

Which of the following assumptions most underlies the chip maker's offer of a discount?

A. Micro Chip expects this discount to help the company retain existing customers and gain
new ones, enabling the firm to survive in the long-term.
B. The government will provide massive technology tax credits to businesses, spurring
them to purchase chips and other related products.
C. The government will not pursue Micro Chip if in fact its behavior in offering a deep
discount amounts to a violation of predatory pricing laws.
D. The decrease in revenue brought about by the reduction in price will be smaller than the
anticipated increase in revenue brought about by the increase

Website Sources: https://www.edufever.com/ Page 8


Critical Ability Set-1
Answer Option D

Q.22: Hunter: Many people blame hunters alone for the decline in Greenrock National
Forest's deer population over the past ten years. Yet clearly, black bears have also
played an important role in this decline. In the past ten years, the forest's protected
black bear population has risen sharply, and examination of black bears found dead
inthe forest during the deer hunting season showed that a number of them had recently
fed on deer.

In the hunter's argument, the portion in bold face plays which of the following roles?

A. It is the main conclusion of the argument.


B. It is a finding that the argument seeks to explain.
C. It introduces a judgment that the argument opposes.
D. It is an explanation that the argument concludes is correct.

Answer Option C

Q.23: 01. Eating beets significantly lowers the risk of cancer, according to an article in a
nutritional magazine. The article refers to a study that found that people who consumed
one or more beets per day were half as likely to be diagnosed with the disease as
people who did not.

Which of the following, if true, most weakens the argument in the magazine article?

A. Another study found that people who consumed one tablespoon of flax seed oil per day
were more than four times less likely to be diagnosed with cancer
B. Participants in the study reported consuming no vegetables other than beets.
C. In another experiment, cancer patients who ate one or more beets per day were no more
likely to recover than those who ate no beets.
D. The participants in the study who ate beets were more likely to exercise regularly than
those who did not eat beets.

Answer Option D

Q.24: A certain automaker aims to increase its market share by deeply discounting its
vehicles' prices for the next several months. The discounts will cut into profits, but
because they will be heavily advertised the manufacturer hopes that they will attract
buyers away from rival manufacturers' cars. In the longer term, the automaker envisions
that customers initially attracted by the discounts may become loyal customers.

In assessing the plan's chances of achieving its aim, it would be most useful to know
which of the following?

Website Sources: https://www.edufever.com/ Page 9


Critical Ability Set-1
A. Whether the automaker's competitors are likely to respond by offering deep discounts on
their own products
B. Whether the advertisements will be created by the manufacturer's current advertising
agency
C. Whether some of the automaker's models will be more deeply discounted than others
D. Whether the automaker will be able to cut costs sufficientlyto maintain profit margins
even when the discounts are in effect

Answer Option A

Q.25: The amount of time it takes for most of a worker's occupational knowledge and
skills to become obsolete has been declining because of the introduction of advanced
manufacturing technology (AMT). Given the rate at which AMT is currently being
introduced in manufacturing, the average worker's old skills become obsolete and new
skills are required within as little as five years.

Which of the following plans, if feasible, would allow a company to prepare most
effectively for the rapid obsolescence of skills described above?

A. The company will develop a program to offer selected employees the opportunity to
receive training six years after they were originally hired.
B. The company will increase its investment in AMT every year for a period of at least five
years.
C. The company will periodically survey its employees to determine how the introduction of
AMT has affected them.
D. The company will ensure that it can offer its employees any training necessary for
meeting their job requirements.

Answer Option D

Q.26: In virtually any industry, technological improvements increase labor productivity,


which is the output of goods and services per person-hour worked. In Parland's
industries, labor productivity is significantly higher than it is in Vergia's industries.
Clearly, therefore, Parland's industries must, on the whole, be further advanced
technologically than Vergia's are.

The argument is most vulnerable to which ofthe following criticisms?

A. It offers a conclusion that is no more than a paraphrase of one of the pieces of


information provided in its support.
B. It presents as evidence in support of a claim information that is inconsistent with other
evidence presented in support of the same claim.
C. It takes one possible cause of a condition to be the actual cause of that condition without
considering any other possible causes.

Website Sources: https://www.edufever.com/ Page 10


Critical Ability Set-1
D. It takes a condition to be the effect of something that happened only after the condition
already existed.

Answer Option C

Q.27: In Washington County, attendance atthe movies isjust large enough for the
cinema operators to make modest profits. The size of the county's population is stable
and is not expected to increase much. Yet there are investors ready to double the
number of movie screens in the county within five years, and they are predicting solid
profits both for themselves and for the established cinema operators.

Which of the following, if true about Washington County, most helps to provide a
justification for the investors' prediction?

A. Over the next ten years, people in their teenage years, the prime moviegoing age, will be
a rapidly growing proportion of the county's population.
B. As distinct from the existing cinemas, most ofthe cinemas being plannedwould be
located in downtown areas, in hopes of stimulating an economic revitalization of those
areas.
C. Spending on video purchases, as well as spending on video rentals, has been
increasing modestly each ear for the past ten years.
D. The average number of screens per cinema is lower among existing cinemas than it is
among cinemas still in the planning stages.

Answer Option A

Q.28: A drug that is highly effective in treating many types of infection can, at present,
be obtained only from the bark of the ibora, a tree that is quite rare in the wild. It takes
the bark of 5,000 trees to make one kilogram of the drug. It follows, therefore, that
continued production of the drug must inevitably lead to the ibora's extinction.

Which of the following, if true, most seriously weakens the argument above?

A. The drug made from ibora bark is dispensed to doctors from a central authority.
B. The drug made from ibora bark is expensive to produce.
C. The leaves of the ibora are used in a number of medical products.
D. The ibora can be propagated from cuttings and grown under cultivation.

Answer Option D

Q.29: The tulu, a popular ornamental plant, does not reproduce naturally, and isonly
bred and sold by specialized horticultural companies. Unfortunately, thetulu is easily
devastated by a contagious fungal rot. The government ministry plans to reassure
worried gardeners by requiring all tulu plants to betested for fungal rot before being

Website Sources: https://www.edufever.com/ Page 11


Critical Ability Set-1
sold. However, infected plants less than 30 weeks old have generally notbuilt up
enough fungal rot in their systems to be detected reliably. And many tulu plants are sold
before they are 24 weeks old.

Which of the following, if performed bythe government ministry, could logically be


expected to overcome the problem with their plan to test for the fungal rot?

A. Releasing a general announcement that tulu plants less than 30 weeks old cannot be
effectively tested for fungal rot
B. Requiring all tulu plants less than 30 weeks old to be labeled as such
C. Researching possible ways to test tulu plants less than 24 weeks old for fungal rot
D. Ensuring that tulu plants not be sold before they are 30 weeks old

Answer Option D

Q.30: For similar carsand comparable drivers, automobile insurance for collision
damage has always costmore in Great port than in Fairmont. Police studies, however,
show thatcars owned by Greatport residents are, on average, slightly less likely to be
involved in a collision than cars in Fairmont. Clearly, therefore, insurance companies
are making a greater profit on collision-damage insurance in Greatportthan in Fairmont.

In evaluating the argument, it would be most useful to compare

A. the level of traffic congestion in Greatport with the level oftraffic congestion in Fairmont
B. the cost of repairing collision damage in Great port with the cost of repairing collision
damage in Fairmont
C. the rates Greatport residents pay for other forms ofinsurance with the rates paid for
similar insurance by residents of Fairmont
D. the condition of Greatport's roads and streets with the condition of Fairmont's roads and
streets

Answer Option B

Website Sources: https://www.edufever.com/ Page 12


Critical Ability Set-1
**This Question Paper Set brought to you by Edufever.com**

For More Question Paper, Placement Paper, Tutorial, Study Materials for job
alerts etc.

Visit: www.edufever.com

Follow Us on:

Website Sources: https://www.edufever.com/ Page 13


Pr
oudl
yPr
esent
s

C
RI
TI
CA
R
L
E
A
R
S
E
O
N
A
I
S
N
G
O
NIN
G
F
orB
ank
in
g,
SS
C,U
PSC
,St
at
ePS
C,R
ai
lwa
y,
Gen
er
al
Comp
et
i-
ti
ona
ndJ
obr
el
at
edex
ams
.

Partners
Critical Reasoning Set-2
Q.1: Last year the rate of inflation was 1.2 percent, but for the current year it has been 4
percent. We can conclude that inflation is on an upward trend and the rate will be still
higher next year.

Which of the following, if true, most seriously weakensthe conclusion above?

A. The inflation figures were computed onthe basis of a representative sampleof economic
data rather than all of the available data.
B. Last year a dip in oil prices brought inflation temporarily below its recent stable annual
level of 4 percent.
C. Increases in the pay of some workers are tied to the level of inflation, and at an inflation
rate of 4 percent or above, these pay raises constitute a force causing further inflation.
D. The 1.2 percent rate of inflation last year represented a 10-year low.

Answer Option B

Q.2: India‘s baffling array of state and national labor laws date to the 1940s: one
provides for the type and number of spittoons in a factory. Another says an enterprise
with more than 100 workers needs government permission to scale back or close. Many
Indian businesses stay small in order to remain beyond the reach of the laws. Big firms
use temporary workers to avoid them. Less than 15% of Indian workers have legal job
security. The new government can sidestep the difficult politics of curbing privileges by
establishing a new, simpler labor contract that gives basic protection to workers but
makes lay-offs less costly to firms. It would apply only to new hires; the small proportion
of existing workers with gold-star protections would keep them.

Which of the following options best summarizes the main idea of the paragraph?

A. More Indian workers can get permanent jobs and legal job security if existing labor laws
are reformed.
B. Effective labor law reform can encourage many Indian businesses to grow to more than
100 workers.
C. Outdated Indian labor laws need to be simplified to provide basic protection to workers
and curb privileges.
D. The difficult politics of curbing privileges can be avoided if the changes in the labor law
only apply to the new hires.

Answer Option C

Q.3: Thyrian lawmaker: Thyria's Cheese Importation Board inspects all cheese
shipments to Thyria and rejects shipments not meeting specified standards. Yet only 1
percent is ever rejected. Therefore, since the health consequences and associated
economic costs of not rejecting that 1 percent are negligible, whereas the board's

Website Sources: https://www.edufever.com/ Page 1


Critical Reasoning Set-2
operating costs are considerable, for economic reasons alone the board should be
disbanded.

Consultant: I disagree. The threat of having their shipments rejected deters many
cheese exporters from shipping substandard product.

The consultant responds to the lawmaker's argument by

A. rejecting the lawmaker's argument while proposing that the standards according to which
the board inspects imported cheese should be raised
B. providing evidence that the lawmaker's argument has significantly overestimated the
cost of maintaining the board
C. objecting to the lawmaker's introducing into the discussion factors that are not strictly
economic
D. pointing out a benefit of maintaining the board, which the lawmaker's argument has
failed to consider

Answer Option D

Q.4: A study published in 2006 by Friedrich Schneider on the world‘s shadow


economies dealt briefly with the ―tax morality‖ of Germans. According to the study, two-
thirds of the Germans surveyed regarded tax evasion as a ―trivial offence,‖ while only
one-third judged stealing a newspaper this way. Indian tax morality is similar, but it
makes a distinction between expatriate illicit money, which is viewed as a serious crime
perpetrated by the very corrupt, and money held within India, which is perceived as a
practical measure.
Which of the following conclusions can be drawn from the above?

(i)Two-thirds of the Germans evade tax and consider it only a trivial offence.
(ii) Stealing a newspaper is a bigger crime in Germany than tax evasion.
(iii) As long as the money is held in India, illicit money is accepted as practical by
Indians.
(iv) Indians regard tax evasion, especially holding illicit money abroad, as a serious
crime.

A. i, ii, iv
B. i and iv
C. only iii
D. iii and iv

Answer Option C

Q.5: The growing popularity of computer-based activities was widely expected to result
in a decline in television viewing, since it had been assumed that people lack sufficient

Website Sources: https://www.edufever.com/ Page 2


Critical Reasoning Set-2
free time to maintain current television-viewing levels while spending increasing
amounts of free time on the computer. That assumption, however, is evidently false: In
a recent mail survey concerning media use, a very large majority of respondents who
report increasing time spent per week using computers report no change intime spent
watching television.

Which of the following would it be most useful to determine in order to evaluate the
argument?

A. Whether a large majority of the survey respondents reported watching television


regularly
B. Whether the amount of time spent watching television is declining among people who
report that they rarely or never use computers
C. Whether the type of television programs a person watches tends to change as the
amount of time spent per week using computers increases
D. Whether the survey respondents' reports of time spent using computers included time
spent using computers at work

Answer Option D

Q.6: Data on planes returning from bombing missions was used to study of the
vulnerability of airplanes to enemy fire. Analyzing the pattern and frequency of hits from
enemy gunfire, it was seen that some parts of planes were hit disproportionately more
often than other parts. How could these planes be optimally reinforced with armor
plating? There were tradeoffs to consider. Every addition of plating added to the weight
of the plane, decreasing its performance. Therefore, reinforcements needed to be
added only to the most vulnerable areas of the planes.

Which of the following can be concluded from the above?

A. The parts hit disproportionately more than the others have to be reinforced as those
received the maximum amount of damage.
B. No conclusion can be drawn as the data set is incomplete. There is no data on the
planes shot down.
C. The parts with the least damage have to be reinforced, as the returning planes have
survived attacks to the most damaged areas.
D. Reinforcements have to be added to all areas of the plane.

Answer Option C

Q.7: Although parapsychology is often considered a pseudoscience, it is in fact a


genuine scientific enterprise, for it uses scientific methods such as controlled
experiments and statistical tests of clearly stated hypotheses to examine the questions
it raises.
Website Sources: https://www.edufever.com/ Page 3
Critical Reasoning Set-2
The conclusion above is properly drawn ifwhich of the following is assumed?

A. If a field of study can conclusively answer the questions it raises, then it is a genuine
science.
B. Since parapsychology uses scientific methods, it will produce credible results.
C. Any enterprise that does not use controlled experiments and statistical tests is not
genuine science.
D. Any field of study that employs scientific methods is a genuine scientific enterprise.

Answer Option D

Q.8: Oklahoma is not perceived as overpopulated because, in spite of a horrendous


drought, it is not facing famine. Famine in Oklahoma is inconceivable because it
receives a fair price for its exports, it has not leased its land to foreign countries, the
poorest of the poor receive a helping hand from the government, and farmers and
ranchers receive federal assistance in times of droughts. It is a lack of these factors in
Horn of Africa, plus political insecurity in Somalia, which explain the famine – not
overpopulation.

Which of the following options best summarizes the main idea of the paragraph?

A. Hunger is caused by too many people pressing against finite resources.


B. In spite of drought and overpopulation, there is no famine in Oklahoma.
C. Overpopulation and famine are not causally related in the Horn of Africa.
D. Famine in the Horn of Africa is not only due to overpopulation but, more importantly, due
to the lack of government assistance and political insecurity.

Answer Option C

Q.9: Studies in restaurants show that the tips left by customers who pay their bill in cash
tend to be larger when the bill is presented on a tray that bears a credit-card logo.
Consumer psychologists hypothesize that simply seeing a credit-card logo makes many
credit-card holders willing to spend more because it reminds them that their spending
power exceeds the cash they have immediately available.

Which of the following, if true, most stronglysupports the psychologists' interpretation of


the studies?

A. The effect noted in the studies is not limited to patrons who have credit cards.
B. Patrons who are under financial pressure from their credit-card obligations tend to tip
less when presented with a restaurant bill on a tray with a credit-card logothan when the
tray has no logo.
C. In virtually all of the cases in the studies, the patrons who paid bills in cash did not
possess credit cards.

Website Sources: https://www.edufever.com/ Page 4


Critical Reasoning Set-2
D. In general, restaurant patrons who paytheir bills in cash leave larger tips than do those
who pay by credit card.

Answer Option B

Q.10: In Dec 2014, the Pew Research Center conducted a survey, asking 1507 people
spread among all 50 states and the District of Columbia , ―What do you think is more
important – to protect the right of Americans to own guns or to control gun ownership?‖
For the first time in more than two decades, a higher percentage (52%) said it was more
important to protect the right of Americans to own guns than to control gun ownership
(46%). The researchers used 1993 as a reference point because it's seen as the height
of gun violence in America; they also noted that gun-related violence fell sharply in the
1990s and more gradually in later years. But Pew also noted that many Americans
didn't seem to agree with the numbers. A survey found that only 12 percent of
respondents thought the gun crime rate was lower than it was in 1993 — and 56
percent thought it was higher.

It can be concluded from this survey result that:

A. Though gun crime is thought to be rising, a majority of people are against stricter
background checks on gun purchases.
B. Most people are not in favor of proposals to restrict gun sales as gun-related violence is
falling.
C. More Americans believe the right to own a gun should be protected, despite perceived
increase in gun crime.
D. With falling gun crime, fewer Americans support controls on gun ownership.

Answer Option C

Q.11:As a construction material, bamboo is as strong as steel and sturdier than


concrete. Moreover, in tropical areas bamboo is a much less expensive construction
material than either steel or concrete and is always readily available. In tropical areas,
therefore, building with bamboo makes better economic sense than building with steel
or concrete, except where land values are high.

Which of the following, if true, most helps to explain the exception noted above?

A. Buildings constructed of bamboo are less likely to suffer earthquake damage than are
steel and concrete buildings.
B. Bamboo is unsuitable as a building material for multi story buildings.
C. In order to protect it from being damaged by termites and beetles, bamboo must be
soaked, at some expense, in a preservative.
D. In some tropical areas, bamboo is used to make the scaffolding that is used during large
construction projects.

Website Sources: https://www.edufever.com/ Page 5


Critical Reasoning Set-2
Answer Option B

Q.12: Championing objectivity in writing—and, more precisely, decrying solipsism,


narcissism, and self-absorption—can of course have legitimate uses. In the era of
Twitter and Facebook, when we are given an infinite supply of blank fields to fill with our
thoughts, we are all encouraged to think we are more interesting than we actually are.
And yet there is a compelling argument to be made, more generally, that journalism‘s
putative standards of objectivity are sometimes wielded to check not subjectivity, per se,
but unwanted subjectivities. The method of testing information – the approach to
evidence- is meant to be objective, not the journalist. The key is in the discipline of the
craft, not the aim.

Which of the following is the writer least likely to agree with?

A. By not taking sides in an argument a journalist can strive to be objective, a worthwhile


aspiration even if it is not perfectly achieved.
B. Objective journalism is not one that is without bias, but one in which bias has to stand up
to evidence and results.
C. All journalism has a point of view and a set of interests it advances.
D. Objectivity in writing is about making the story more than just about the writer.

Answer Option A

Q.13: In the country of Russia, the past two years' broad economic recession has
included a business downturn in clothing trade, where sales are down by about 7
percent as compared to two years ago. Clothing wholesalers have found, however, that
the proportion of credit extended to retailers that was paid off on time fell sharply in the
first year of the recession but returned to its prerecession level in the second year.

Which of the following, if true, most helps to explain the change between the first and
the second year of recession in the proportion of credit not paid off on time?

A. The total amount of credit extended to retailers by clothing wholesalers increased


between the first year of the recession and the second year.
B. Between the first and second years of the recession, clothing retailers in Russia saw
many of their costs, rent and utilities in particular, increase.
C. Of the considerable number of clothing retailers in Russia who were having financial
difficulties before the start of the recession, virtually all were forced to go out of business
during its first year.
D. Clothing retailers in Russia attempted to stimulate sales in the second year of the
recession by discounting merchandise.

Answer Option C

Website Sources: https://www.edufever.com/ Page 6


Critical Reasoning Set-2
Q.14: Contrary to popular belief, the idea of evolution didn‘t originate with Darwin, but
was around for decades before he came along. His accomplishment was to come up
with a workable scheme by which it likely occurred. Darwin, it must be said, had unusual
exposure to the enormous diversity of life on earth for a man living in his time, through
his voyage on the Beagle. However, his eureka moment came not through studying
biology, but by reading the paper of an economist, Thomas Malthus, which showed that
populations grow faster than the resources to sustain them. It was then that Darwin
realized that only those best adapted to their environment would survive and pass on
their traits to offspring.

What is the main idea of this paragraph?

A. It is impossible to come up with big insights without crossing domains.


B. The idea of evolution did not originate with Darwin.
C. It is Malthus, not Darwin, who should be given credit for the theory of evolution.
D. It was the connections that Darwin uncovered more than the facts themselves that made
his work important.

Answer Option D

Q.15: Museums that house Renaissance oil paintings typically store them in
environments that are carefully kept within narrow margins of temperature and humidity
to inhibit any deterioration. Laboratory tests have shown that the kind of oil paint used in
these paintings actually adjusts to climatic changes quite well. If, as some museum
directors believe, paint is the most sensitive substance in these works, then by relaxing
the standards for temperature and humidity control, museums can reduce energy costs
without risking damage to these paintings. Museums would be rash to relax those
standards, however, since results of preliminary tests indicate that gesso, a compound
routinely used by Renaissance artists to help paint adhere to the canvas, is unable to
withstand significant variations in humidity.

In the argument above, the two portions in boldface play which of the following roles?

A. The first is an objection that has been raised against the position taken by the argument;
the second is the position taken by the argument.
B. The first is the position taken by the argument; the second is the position that the
argument calls into question.
C. The first is a judgment that has been offered in support of the position that the argument
calls into question; the second is a circumstance on which that judgment is, in part,
based.
D. The first is a judgment that has been offered in support of the position that the argument
calls into question; the second is that position.

Website Sources: https://www.edufever.com/ Page 7


Critical Reasoning Set-2

Answer Option D

Q.16: It has been argued by behavioral economists in all earnestness that poverty
shapes mindsets. From here, it is a hop, skip, and jump to holding that the poor are
poor because their poverty prevents them from thinking and acting in ways that can take
them out of poverty. When behaviorist economics speaks of poverty as a ―cognitive tax‖,
it writes ‗action‘ — the political agency of the poor — out of the equation. In such a
case, the focus as well as the onus of poverty-alleviation would shift from the state —
from macroeconomic policy, from having to provide employment, health and education
— to changing the behavior of the poor. The structural causes of poverty — rising
inequality and unemployment — as well as the behavior of the owners of capital are
evicted from the poverty debate, and no longer need be the focus of public policy.

Which of these options best summarizes the given paragraph?

A. Where decisions of the poor tend to be flawed from an economic point of view,
behavioral economists believe that governments can intervene with policies aimed at
nudging the poor towards the right decision.
B. By shifting the burden of poverty alleviation from the state onto the poor themselves,
behavioral economists are ignoring both the structural causes of poverty as well as the
behavior of the wealthy.
C. Given that poverty diminishes political agency and shapes mindsets, insights into how
poverty affects behavior could have implications in public policy.
D. The focus of public policy ought to be in providing employment, health and education,
rather than addressing rising inequality and changing the behavior of the poor.

Answer Option B

Q.17: Hotco oil burners, designed to be used in asphalt plants, are so efficient that
Hotco will sell one to the Clifton Asphalt plant for no payment other than the cost
savings between the total amount the asphalt plant actually paid for oil using its former
burner during the last two years and the total amount it will payfor oil using the Hotco
burner during the next two years. On installation, the plant will make an estimated
payment, which will be adjusted after two years to equal the actual cost savings.

Which of the following, if it occurred, would constitute a disadvantage for Hotco of the
plan described above?

A. Another manufacturer's introduction to the market of a similarly efficient burner


B. The Clifton Asphalt plant's need for more than one new burner
C. Very poor efficiency in the Clifton Asphalt plant's old burner
D. A steady increase in the price of oil beginning soon after the new burner is installed

Website Sources: https://www.edufever.com/ Page 8


Critical Reasoning Set-2
Answer Option D

Q.18: Many scientific studies have found links between genius and mental illness,
particularly bipolar disorder in which patients have violent mood swings between elation
and depression. In one interesting Swedish study, 700,000 Swedes had intelligence
tests at age 16 and again 10 years later. Those who scored well were four times more
likely than the others to have developed bipolar disorder. The US neurologist James
Fallon came up with a convincing argument based on his own findings in the field: the
brain area involved in mood swings is the same area where creativity is born. This may
explain why some people can draw previously unseen connections among ideas,
images, shapes and the like.

Which of the following is inferred from the above?

A. Those with bipolar disorder are likely to be highly intelligent.


B. Most people who excel in creativity are likely suffering from a mental problem.
C. Often there is a correlation between mental illness and genius.
D. Mental disorders give birth to genius.

Answer Option C

Q.19: An experiment was done in which human subjects recognize a pattern within a
matrix of abstract designs and then select another design that completes that pattern.
The results ofthe experiment were surprising. The lowest expenditure of energy in
neurons in the brain wasfound in those subjects who performed most successfully in the
experiments.

Which of the following hypotheses best accounts for the findings of the experiment?

A. The neurons of the brain react less when a subject is trying to recognize patterns than
when the subject is doing other kinds of reasoning.
B. Those who performed best in the experiment experiencedmore satisfaction when
working with abstract patterns than did those who performed less well.
C. People who are better at abstract pattern recognition have more energy-efficient neural
connections.
D. The energy expenditure of the subjects' brains increases when a design that completes
the initially recognized pattern is determined.

Answer Option C

Q.20: One way to judge the performance of a company is to compare it with other
companies. This technique, commonly called "benchmarking," permits the manager of a
company to discover better industrial practices and can provide a justification for the
adoption of good practices.

Website Sources: https://www.edufever.com/ Page 9


Critical Reasoning Set-2
Any of the following, if true, is a valid reason for benchmarking the performance of a
company against companies with which it is not in competition ratherthan against
competitors EXCEPT:

A. Comparisons with competitors are most likely to focus on practices that the manager
making the comparisons already employs.
B. Getting "inside" information about the unique practices of competitors is particularly
difficult.
C. Since companies that compete with each other are likely to have comparable levels of
efficiency, only benchmarking against non competitors is likely to reveal practices that
would aid in beating competitors.
D. Much of the success of good companies is due to their adoption of practices that take
advantage of the special circumstances of their products or markets.

Answer Option D

Q.21: Any reporter worthy of the name would no sooner fiddle with direct quotes than a
reputable photojournalist would alter his or her picture. News photographs are the
equivalent of direct quotations and therefore are sacrosanct. To be sure, just as a writer
can, in the interest of brevity or impact, choose which quotes to use in a story, so can a
news photographer or picture editor crop out dead space in a news photo, or use the
electronic equivalent of dodging or burning in to make a picture reproduce better. In this,
I am reminded of what a Washington Times shooter once told me. On a computer
outside the paper's darkroom, she said, there was plastered this flat admonition and
warning: "If you can't do it in the darkroom, don't do it here".

Which of the following is the author least likely to agree with?

A. In photojournalism, editing news photographs by truncating dead space is licit.


B. The key elements of a news photograph, like the key words in a direct quote, are off
limits to manipulation.
C. Journalists of repute choose amongst, but do not distort direct quotations.
D. It is permissible for a photojournalist to alter a news photo in the interest of brevity or
impact.

Answer Option D

Q.22: For a trade embargo against a particularcountryto succeed, a high degree of both
international accord and ability to prevent goods from entering or leaving that country
must be sustained. Atotal blockade of Patria's ports is necessary to an embargo, but
such an action would be likely to cause international discord over the embargo.

The claims above, iftrue, most strongly support which of the following conclusions?

Website Sources: https://www.edufever.com/ Page 10


Critical Reasoning Set-2
A. The balance of opinion is likely to favor Patria inthe event of a blockade.
B. As long as international opinion is unanimously against Patria, a trade embargo is likely
to succeed.
C. A naval blockade of Patria's ports would ensure that no goods enter or leave Patria.
D. Any trade embargo against Patria would be likely to fail at some time.

Answer Option D

Q.23: A holistic reading of the current state of our constitutional jurisprudence would
demonstrate that the right to privacy is firmly embedded in our constitutional scheme as
a non-negotiable imperative that owes no apology to a myopic view of our republican
charter. Indeed, considering the fundamental principles of the nation as ―not rules for
the passing hour, but principles for an expanding future‖, the apex court, as the ultimate
arbiter of constitutional conscience, has given fundamental rights their meaning in new
settings consistent with the aspirations of our people. This is so that we may have a
‗living constitution‘ which can protect, preserve and defend sacrosanct libertarian values
that remain the bedrock of the Republic and constitute the core of the Constitution.
Rather than deny us our constitutional right , the Union Government ought to enact a
privacy legislation to clearly define the rights of citizens consistent with the promise of
the Constitution.

Which of the following is the author most likely to agree with?

A. Our republican charter has a myopic view of the right to privacy


B. The Supreme Court has been rigid in its interpretation of the Constitution
C. The right to privacy is rooted in our constitutional scheme.
D. A new privacy legislation has to be defined as the right to privacy is not dealt with in the
Constitution.

Answer Option C

Q.24: In an attempt to encourage ―livelier‖ writing, some teachers want children to stop
using words like ‗said‘, which doesn‘t have any emotion. The assumption here is that
emotion is a desirable quality in every word of a sentence, and that a rich word is
always more appropriate than a plain one. You don‘t have to invoke Hemingway, who
made a fetish of plain words, to recognize that successful writing modulates the
lavishness of its diction for effect, rather than cranking the dial all the way to maximum
floridity and leaving it there.

What is the main idea of this paragraph?

A. Successful writers use only plain, unemotional words.


B. Emotion is not a desirable quality in every sentence.

Website Sources: https://www.edufever.com/ Page 11


Critical Reasoning Set-2
C. Avoiding words like ―said‖ helps children improve their writing.
D. Good writers use rich words in moderation, for effect.

Answer Option D

Q.25: The image of an oral telling may be caught on paper, film or in digital format, but
recordings are not the word shared live. The presence of teller and audience, and the
immediacy of the moment are not fully captured by any form of technology. Unlike the
insect frozen in amber, a told story is alive. It always changes from one telling to the
next depending on the voice and mood of the storyteller, the place of its telling and the
response of the audience. The story breathes with the teller‘s breath.

Which of the options below conveys the main idea of the paragraph best?

A. Unlike stories in paper, film and digital formats, a told story is alive, and hence more
potent.
B. The immediacy and intimacy of live storytelling cannot be captured by recordings.
C. Technology and oral storytelling are separated by a deep divide and one cannot take the
place of the other.
D. The living human presence experienced through oral tradition brings out the true power
of stories.

Answer Option B

Q.26: Euphemisms in use seem to change every generation or so – a tendency towards


which we often roll our eyes. But the fact is that a word is always redolent of various
associations and metaphorical extensions beyond its core meaning. Indeed, a word is
like a bell tone, with a central pitch seasoned by overtones. As the tone fades away, the
overtones can hang in the air. Words then bias as equivalents to the overtones. As we
move on the euphemism treadmill then, from ‗crippled‘ to ‗handicapped‘ to ‗disabled‘ to
‗differently abled‘, we acknowledge the eternal gulf between language and opinion. In a
linguistically mature society, we should expect that the terms we introduce to help us
kick off new ways of thinking will require periodic replacement, like tyres.

What is the main idea of this paragraph?

A. All words wear out with use, like tyres, and need to be replaced periodically with more
meaningful equivalents.
B. The eternal quest for euphemisms to bridge the gap between language and opinion is
tedious.
C. We must accept the euphemism treadmill as an essential part of linguistic life in a
civilized society.
D. Euphemisms help us avoid the trap of thinking too much into the meanings of words.

Website Sources: https://www.edufever.com/ Page 12


Critical Reasoning Set-2
Answer Option C

Q.27: When people start debating on whether the effects of the Internet are good or
bad, it is the content they wrestle over. What both enthusiasts and skeptics miss is the
fact that as a window onto the world, and onto ourselves, the Internet shapes what we
see and how we see it, and eventually, if we use it enough, changes who we are as
individuals and as a society. It controls the scale and form of human association and
delivers a new form of human being, whose qualities are suited to it. As Marshall
McLuhan, author of ‗Understanding Media: The Extensions of Man‘ wrote, the content of
the medium is just the juicy bit of meat carried by the burglar to distract the watchdog of
the mind.

Which of the following options best summarizes the main idea of this paragraph?

A. The debate on the Internet‘s content is pointless; content does not matter.
B. The Internet is neither good nor bad; it is the way it is used that determines its value.
C. The focus on the Internet‘s content can blind us to its effects on our thought and action.
D. The Internet distracts our mind by flooding us with content.

Answer Option C

Q.28: Many students, especially those who are poor, intuitively know what the schools
do for them. They school them to confuse process and substance. Once these become
blurred, a new logic is assumed: the more treatment there is, the better are the results;
or, escalation leads to success. The pupil is thereby schooled to confuse teaching with
learning, grade advancement with education, a diploma with competence, and fluency
with the ability to say something new. His imagination is schooled to accept service in
place of value. Medical treatment is mistaken for health care, social work for the
improvement of community life, police protection for safety, military poise for national
security, the rat race for productive work. Health, learning, dignity, independence, and
creative endeavor are defined as little more than the performance of the institutions
which claim to serve these ends, and their improvement is made to depend on
allocating more resources to the management of hospitals, schools, and other agencies
in question.

What is the main idea of this paragraph?

A. Allocating more resources to health, learning, dignity, independence, and creative


endeavor is not likely to increase their quality.
B. Schooling doctors thinking by teaching students to identify the process with the results.
C. As a society we often mistake grades and diplomas for competence, medical treatment
for healthcare, police protection for safety and the rat race for productive work.

Website Sources: https://www.edufever.com/ Page 13


Critical Reasoning Set-2
D. When it comes to schooling, less is more: the less the treatment there is, better the
results

Answer Option B

Q.29: As it mindlessly implements its procedures, an algorithm is supposed to be


devoid of bias. When an algorithm rejects a loan application or sets the price for an
airline flight, it is deemed impersonal and unbending. Yet, an algorithm is not a scientific
concept. It is a system, like a military chain of command. It takes knowhow, calculation
and creativity to make a system work properly. And some systems, like some armies,
are much more reliable than others. For, a system is a human artefact, not a
mathematical truism.

Which of the following options best summarizes the main idea of this paragraph?

A. As they are man-made, algorithms are subject to human fallibility.


B. Algorithms are far from being precise, and invariably biased.
C. Not all algorithms work properly, as they are rarely vetted with scientific rigour.
D. Algorithms are unreliable as they encode human prejudice into automated systems.

Answer Option A

Q.30: On the rare occasions that they see it fit to direct their attention toward older
adults, most tech innovators instinctively reduce their target users down to their medical
issues. Consumed with the obvious problems of old age—issues like hearing, mobility,
medication management—they fail to consider the other things someone of any age
would want. Concerns about self-image or style, crucial considerations for every other
age group, are frequently seen as frivolous, and come only as an afterthought.
Meanwhile, for many older adults, the higher-level desire to look and feel a certain way
can overpower base-level physiological needs such as the ability to hear or see clearly.
And so, for every older adult who has a hearing aid, an emergency-response necklace,
or one of those cell phones with the huge rubber buttons, there are legions avoiding
these potentially lifesaving devices.

Which of the following options best summarizes the main idea of this paragraph?

A. For older adults, as with every other age group, psychological needs trump physiological
needs.
B. Tech innovators rarely direct their attention to older adults and do not understand the
needs of this age group.
C. That older adults are conscious of self-image and style must be taken into account by
tech innovators designing products for them.
D. Lifesaving devices for older adults will be adopted more widely if they are stylishly
designed.
Website Sources: https://www.edufever.com/ Page 14
Critical Reasoning Set-2
Answer Option C

**This Question Paper Set brought to you by Edufever.com**

For More Question Paper, Placement Paper, Tutorial, Study Materials for job
alerts etc.

Visit: www.edufever.com

Follow Us on:

Website Sources: https://www.edufever.com/ Page 15


Pr
oudl
yPr
esent
s

C
RI
TI
CA
R
L
E
A
R
S
E
O
N
A
I
S
N
G
O
NIN
G
F
orB
ank
in
g,
SS
C,U
PSC
,St
at
ePS
C,R
ai
lwa
y,
Gen
er
al
Comp
et
i-
ti
ona
ndJ
obr
el
at
edex
ams
.

Partners
Critical Reasoning Set-3
Q.1: In times when expectations can change at the drop of a hat, flexibility is as
important as speed. However, in their zeal for efficiency, some organizations have
engineered out all the slack they can from their businesses at the expense of agility. As
Tom DeMarco puts it in his book Slack, „An organization that can accelerate but not
change direction is like a car that can speed up but not steer. In the short run, it makes
lots of progress in whatever direction it happened to be going. In the long run, it‟s just
another road wreck.‟

Which of the options below conveys the main idea of the paragraph best?

A. Only organizations that are not very efficient can adapt to change well.
B. Too little slack in organizations is as bad for innovation as too much slack.
C. Reducing slack to zero can cost businesses their adaptability to change.
D. Business processes without slack slow down an organization‟s progress.

Answer Option C

Q.2: While society is chipping away at giving girls broader access to life‟s possibilities, it
isn‟t presenting boys with a full continuum of how they can be in the world. To carve out
a masculine identity requires whittling away everything that falls outside the norms of
boyhood. At the earliest ages, it‟s about external signifiers like favorite colors, TV
shows, and clothes. But later, the paring knife cuts away intimate friendships, emotional
range, and open communication.

Which of the following options puts forth the main idea of this paragraph best?

A. Boys are put in a straitjacket in order to conform to the popular, narrow view of
masculine identity.
B. It is lopsided to approach gender equality by focusing only on girls‟ empowerment.
C. It is a societal loss that boys are not allowed free self-expression.
D. Boys are forced to limit their emotional range in order to carve out a masculine identity.

Answer Option A

Q.3: A law requiring companies to offer employees unpaid time off to care for their
children will harm the economic competitiveness of our nation‟s businesses. Companies
must be free to set their own employment policies without mandated parental-leave
regulations. Which of the following, if true, would most seriously weaken the conclusion
of the argument above?

A. Many businesses in this country already offer employees some form of parental leave.
B. A parental-leave law will serve to strengthen the family as a social institution in this
country.
C. In most polls, a majority of citizens say they favor passage of a parental-leave law.

Website Sources: https://www.edufever.com/ Page 1


Critical Reasoning Set-3
D. Some of the countries with the most economically competitive businesses have strong
parental-leave regulations.

Answer Option D

Q.4: Studies of fatal automobile accidents reveal that, in the majority of cases in which
one occupant of an automobile is killed while another survives, it is the passenger, not
the driver, who is killed. It is ironic that the innocent passenger should suffer for the
driver‟s carelessness, while the driver often suffers only minor injuries or none at all.

Which of the following is an assumption underlying the reasoning in the passage


above?

A. In most fatal automobile accidents, the driver of a car in which an occupant is killed is at
fault.
B. Drivers of automobiles are rarely killed in auto accidents.
C. Auto safety experts should increase their efforts to provide protection for those in the
passenger seats of automobiles.
D. Most deaths in fatal automobile accidents are suffered by occupants of cars rather than
by pedestrians.

Answer Option A

Q.5: Take „the birthday problem‟, for instance. It simply asks: how many people would
you need to get into the same room in order to statistically assure that at least two share
the exact birth month and day? Given that there are 365 days in a non-leap year, and
that most people you know probably don‟t have the same birthday, you might
reasonably suppose that you‟d need quite a high number to find an exact match.
Hundreds, perhaps, and even then you‟d be lucky to find two people with the same birth
month and day. Statistically, however, you need only 23 people in the room for a greater
than 50 per cent (hence „statistically probable‟) chance of finding two people with the
exact same birth month and day. For a 99.9 per cent chance, you need only 70 people.

Which of the following options puts forth the main idea of this paragraph best?

A. Often, coincidences are given a significance disproportionate to their relative


commonness.
B. Coincidences can be explained by probabilistic and statistical reasoning.
C. Coincidences aren't as low probability as commonly thought.
D. Coincidences are bound to happen and are void of greater meaning.

Answer Option A

Website Sources: https://www.edufever.com/ Page 2


Critical Reasoning Set-3
Q.6: Are you still reading the other newspaper in town? Did you know that the Daily
Bugle is owned by an out-of-town business syndicate that couldn‟t care less about the
people of Gotham City? Read the Daily Clarion, the only real voice of the people of
Gotham City!

Which of the following most directly refutes the argument raised in the advertisement
above?

A. Over half of the advertising revenues of the Daily Clarion come from firms whose
headquarters are located outside of Gotham City.
B. The Daily Clarion usually devotes more of its pages to out-of-town news than does the
Daily Bugle.
C. The editor-in-chief and all the other members of the editorial staff of the Daily Bugle have
lived and worked in Gotham City for ten years or more.
D. The Daily Bugle has been published in Gotham City for a longer time than has the Daily
Clarion.

Answer Option C

Q.7: Paragraph: In order to determine automobile insurance premiums for a driver,


insurance companies calculate various risk factors; as the risk factors increase, so does
the premium. Certain factors, such as the driver‟s age and past accident history, play an
important role in these calculations. Yet, these premiums should also increase with the
number of miles that the driver drives. After all, a person‟s chance of being involved in a
mishap increases in proportion to the number of times that person drives.

Which one of the following, if true, most undermines the argument?

A. People who drive infrequently are more likely to be involved in accidents that occur on
small roads than in highway accidents.
B. People who drive infrequently are less likely to follow rules for safe driving than are
people who drive frequently.
C. People who drive infrequently are less likely to violate local speed limits than are people
who drive frequently.
D. People who drive frequently are more likely to make long-distance trips in the course of
a year than are people who drive infrequently.

Answer Option B

Q.8: Anthropologist: Climate and geology determine where human industry can be
established. Drastic shifts in climate always result in migrations, and migrations bring
about the intermingling of ideas necessary for rapid advances in civilization.

Website Sources: https://www.edufever.com/ Page 3


Critical Reasoning Set-3
The anthropologist's statements, if true, most strongly support which one of the
following?

A. Climate is the primary cause of migration


B. All shifts in climate produce a net gain in human progress.
C. A population remains settled only where the climate is fairly stable.
D. Populations settle in every place where human industry can be established.

Answer Option C

Q.9: The claim that insurance premiums should increase as the frequency with which a
driver drives increases plays which one of the following roles in the argument?

A. A premise of the argument


B. The conclusion of the argument
C. Evidence offered in support of one of the premises
D. An assertion phrased to preclude an anticipated objection

Answer Option B

Q.10: Alba: I don‟t intend to vote for Senator Frank in the next election. She is not a
strong supporter of the war against crime.
Tam: But Senator Frank sponsored the latest anti-crime law passed by the Senate.
Alba: If Senator Frank sponsored it, it can‟t be a very strong anti-crime law.

Which of the following identifies the most serious logical flaw in Alba‟s reasoning?

A. The facts she presents do not support her conclusion that Senator Frank is soft on
crime.
B. She assumes without proof that crime is the most important issue in the upcoming
election.
C. She argues in a circle, using an unsupported assertion to dismiss conflicting evidence.
D. She attacks Senator Frank on personal grounds rather than on the merit as a political
leader.

Answer Option C

Q.11: Paragraph: At an enormous research cost, a leading chemical company has


developed a manufacturing process for converting wood fibers into a plastic. According
to the company, this new plastic can be used for, among other things, the hulls of small
sailboats. But what does the company think sailboat hulls used to be made of? Surely
the mania for high technology can scarcely go further than this.

Website Sources: https://www.edufever.com/ Page 4


Critical Reasoning Set-3
(1). The author‟s opinion of the manufacturing process described in the passage is
based primarily on the fact that

A. The research costs of developing the process outweigh any savings possible from the
use of the plastic
B. A small sailboat is not normally regarded as a high-tech product
C. Hulls for small sailboats can be made from wood without converting it into plastic
D. Many other spheres of human activity are in far greater need of technological research

Answer Option C

(2). Which of the following, if true, would most seriously weaken the author‟s conclusion?

A. The wood used in producing the plastic is itself in increasingly short supply.
B. The plastic produced by the process is considerably lighter, stronger, and more
watertight than wood.
C. The cost of the manufacturing process of the plastic increases the cost of producing a
sailboat hull by 10 to 15 percent.
D. Much of the cost of the research that developed the new process will be written off for
tax purposes by the chemical company.

Answer Option B

Q.13: The earth‟s resources are being depleted much too fast. To correct this, the
United States must keep its resource consumption at present levels for many years to
come. The argument above depends on which of the following assumptions?

A. Per capita resource consumption in the United States is at an all-time high.


B. The United States uses more resources than any other country.
C. Curbing U.S. resource consumption will significantly retard world resource depletion.
D. The United States imports most of the resources it uses.

Answer Option C

Q.14: Because people are generally better at detecting mistakes in other‟s work than in
their own, a prudent principle is that one should always have one‟s own work checked
by someone else.Which one of the following provides the best illustration of the principle
above?

A. The best elementary school math teachers are not those for whom math was always
easy. Teachers who had to struggle through math themselves are better able to explain
math to students.
B. One must make a special effort to clearly explain one‟s views to someone else; people
normally find it easier to understand their own views than to understand others‟ views.

Website Sources: https://www.edufever.com/ Page 5


Critical Reasoning Set-3
C. Juries composed of legal novices, rather than panels of lawyers, should be the final
arbiters in legal proceedings. People who are not legal experts are in a better position to
detect good legal arguments by lawyers than are other lawyers.
D. People should always have their writing proofread by someone else. Someone who
does not know in advance what is meant to be said is in a better position to spot
typographical errors.
E.

Answer Option D

Q.15: Below is an excerpt from a letter that was sent by the chairman of a corporation to
the stockholders.

A number of charges have been raised against me, some serious, some trivial.
Individuals seeking to control the corporation for their own purposes have demanded
my resignation. Remember that no court of law in any state has found me guilty of any
criminal offense whatsoever. In the Indian tradition, as you know, an individual is
considered innocent until proven guilty. Furthermore, as the corporation‟s unbroken six-
year record of growth will show, my conduct of my official duties as chairman has only
helped enhance the success of the corporation, and so benefited every stockholder.

Which of the following can be properly inferred from the excerpt?

A. Any misdeeds that the chairman may have committed were motivated by his desire to
enhance the success of the corporation.
B. The chairman is innocent of any criminal offense.
C. The corporation has expanded steadily over the past six years.
D. Any legal proceedings against the chairman have resulted in his acquittal.

Answer Option D

Q.16: Recent research indicates that increased consumption of fruits and vegetables by
middle-aged people reduces their susceptibility to stroke in later years. The researchers
speculate that this may be because fruits and vegetables are rich in folic acid. Low
levels of folic acid are associated with high levels of homocysteine, an amino acid that
contributes to blocked arteries. Which one of the following statements is most strongly
supported by the information above?

A. An increased risk of stroke is correlated with low levels of homocysteine.


B. A decreased risk of stroke is correlated with increased levels of folic acid.
C. An increased propensity for blocked arteries is correlated with decreased levels of
homocysteine.
D. A decreased propensity for blocked arteries is correlated with low levels of folic acid.

Website Sources: https://www.edufever.com/ Page 6


Critical Reasoning Set-3
Answer Option B

Q.17: In the years since the city of London imposed strict air-pollution regulations on
local industry, the number of bird species seen in and around London has increased
dramatically. Similar air-pollution rules should be imposed in other major cities.

A. In most major cities, air-pollution problems are caused almost entirely by local industry.
B. Air-pollution regulations on industry have a significant impact on the quality of the air.
C. The air-pollution problems of other major cities are basically similar to those once
suffered by London.
D. An increase in the number of bird species in and around a city is desirable.

Answer Option A

Q.18: Mr. Janeck: I don‟t believe Stevenson will win the election for governor. Few
voters are willing to elect a businessman with no political experience to such a
responsible public office. Ms. Siuzdak: You‟re wrong. The experience of running a major
corporation is a valuable preparation for the task of running a state government. M.
Siuzdak‟s response shows that she has interpreted Mr. Janeck‟s remark to imply which
of the following?

A. Mr. Janeck considers Stevenson unqualified for the office of governor.


B. No candidate without political experience has ever been elected governor of a state.
C. Mr. Janeck believes that political leadership and business leadership are closely
analogous.
D. Voters generally overestimate the value of political experience when selecting a
candidate.

Answer Option A

Q.19: The ancient city of Cephesa was not buried by an eruption of Mt. Amnos in A.D.
310, as some believe. The eruption in the year 310 damaged the city, but it did not
destroy it. Cephesa survived for another century before it finally met its destruction in
another eruption around A.D. 415.

Which of the following, if true, would most strengthen the author‟s claim that the city of
Cephesa was not buried by the eruption of Mt. Amnos in A.D. 310?

A. The city of Cephesa is mentioned in a historical work known to have been written in A.D.
400.
B. Coins bearing the image of an emperor who lived there around A.D. 410 have been
discovered in the ruins of Cephesa, which were preserved by the cinders and ashes that
buried the city.

Website Sources: https://www.edufever.com/ Page 7


Critical Reasoning Set-3
C. Geological evidence shows that the eruption of Mt. Amnos in A.D. 415 deposited a 10-
foot-thick layer of lava on the city of Cephesa.
D. A historical work written in A.D. 430 refers to the eruption of Mt. Amnos in A.D. 415.

Answer Option B

Q.20: At one time, European and Japanese companies tried to imitate their American
rivals. Today, American appliance manufacturers import European scientists to lead
their research staffs; American automakers design cars that mimic the styling of
German, Italian, and French imports; and American electronics firms boast in their
advertising of “Japanese-style” devotion to quality and reliability. In the world of high
technology, America has lost the battle for international prestige.Each of the following
statements, if true, would help to support the claim above EXCEPT:

A. An American camera company claims in its promotional literature to produce cameras


“as fine as the best Swiss imports.”
B. An American maker of stereo components designs its products to resemble those of a
popular Japanese firm.
C. An American manufacturer of video games uses a brand name chosen because it
sounds like a Japanese word.
D. An American maker of frozen foods advertises its dinners as “Real European-style
entrees prepared by fine French and Italian chefs.”

Answer Option D

Q.21: Reva: Using extraneous incentives to get teenagers to change their attitude
toward school and schoolwork won‟t work. Take the program in West Virginia, for
instance, where they tried to reduce their dropout rate by revoking the driving licenses
of kids who left school. The program failed miserably.

Anne: It‟s true that the West Virginia program failed, but many schools have devised
incentive programs that have been very successful in improving attendance and
reducing discipline problems.

According to Anne, the weak point in Reva‟s claim is that it

A. fails to consider the possibility that the majority of potential dropouts in West Virginia do
not have driving licenses
B. doesn‟t provide any exact figures for the dropout rate in West Virginia before and during
the program
C. ignores a substantial body of evidence showing that parents and employers have been
using extrinsic incentives with positive results for years
D. is based on a single example, the incentive program in West Virginia, which may not be
typical

Website Sources: https://www.edufever.com/ Page 8


Critical Reasoning Set-3
Answer Option D

Q.22: Johnson is on firm ground when he asserts that the early editors of Dickinson‟s
poetry often distorted her intentions. Yet Johnson‟s own, more faithful, text is still guilty
of its own forms of distortion. To standardize Dickinson‟s often indecipherable
handwritten punctuation by the use of the dash is to render permanent a casual mode of
poetic phrasing that Dickinson surely never expected to see in print. It implies that
Dickinson chose the dash as her typical mark of punctuation when, in fact, she
apparently never made any definitive choice at all. Which of the following best
summarizes the author‟s main point?

A. Although Johnson is right in criticizing Dickinson‟s early editors for their distortion of her
work, his own text is guilty of equally serious distortions.
B. Johnson‟s use of the dash in his text of Dickinson‟s poetry misleads readers about the
poet‟s intentions.
C. Because Dickinson never expected her poetry to be published, virtually any attempt at
editing it must run counter to her intentions.
D. Although Johnson‟s attempt to produce a more faithful text of Dickinson‟s poetry is well-
meaning, his study of the material lacks sufficient thoroughness.

Answer Option A

Q.23: In many surveys, American consumers have expressed a willingness to spend


up to 10 percent more for products that are ecologically sound. Encouraged by such
surveys, Bleach-O Corporation promoted a new laundry detergent, Bleach-O Green, as
safer for the environment. Bleach-O Green cost 5 percent more than typical detergents.
After one year, Bleach-O Green had failed to capture a significant share of the
detergent market and was withdrawn from sale.

Which of the following questions is LEAST likely to be relevant in determining the


reasons for the failure of Bleach-O Green?

A. How effective as a detergent was Bleach-O Green?


B. How many other detergents on the market were promoted as safe for the environment?
C. How much more did Bleach-O Green cost to manufacture than ordinary detergents?
D. To what extent did consumers accept the validity of Bleach-O Green advertised and
promoted to consumers?

Answer Option C

Q.24: Ronald: According to my analysis of the national economy, housing prices should
not increase during the next six months unless interest rates drop significantly.

Website Sources: https://www.edufever.com/ Page 9


Critical Reasoning Set-3
Mark: I disagree. One year ago, when interest rates last fell significantly, housing prices
did not increase at all.

It can be inferred from the conversation above that Mark has interpreted Ronald‟s
statement to mean that

A. housing prices will rise only if interest rates fall


B. if interest rates fall, housing prices must rise
C. interest rates and housing prices tend to rise and fall together
D. interest rates are the only significant economic factor affecting housing prices

Answer Option B

Q.25: The burden of maintaining the U.S. highway system falls disproportionately on the
trucking industry. Trucks represent only about 10 percent of the vehicles on U.S. roads.
Yet road use taxes assessed on trucks amount to almost half the taxes paid for highway
upkeep and repair.

Which of the following, if true, would most weaken the argument above?

A. The trucking industry has enjoyed record after-tax profits in three of the past four years.
B. Because of their weight, trucks cause over 50 percent of the damage sustained by
highway surfaces each year.
C. Without an economically viable trucking industry, the cost of goods in the United States
would rise significantly.
D. Road use taxes paid by trucking companies have decreased by 3 percent over the past
five years.

Answer Option B

Q.26: As an experienced labor organizer and the former head of one of the nation‟s
most powerful labor unions, Grayson is an excellent choice to chair the new council on
business-labor relations.

Which of the following, if true, would most strengthen the conclusion above?

A. The new council must have the support of the nation‟s labor leaders if it is to succeed.
B. During his years as a labor leader, Grayson established a record of good relations with
business leaders.
C. An understanding of the needs and problems of labor is the only qualification necessary
for the job of chairing the new council.
D. Most of the other members of the new council will be representatives of business
management interests.

Answer Option C

Website Sources: https://www.edufever.com/ Page 10


Critical Reasoning Set-3
Q.27: The upcoming presidential election in the West African republic of Ganelon is of
grave concern to the U.S. State Department. Ganelon presently has strong political and
military ties to the United States. However, the Socialist party is widely expected to win
the election, leading to fears that Ganelon will soon break away from the pro-American
bloc and adopt a nonaligned or openly anti-American stance.

Which of the following is an assumption made in the passage above?

A. A Socialist party government in Ganelon is more likely to oppose the United States than
is a non-Socialist party government.
B. The people of the United States recognize their nation‟s interest in the political stability of
West Africa.
C. A weakening of U.S. political ties with Ganelon could have serious consequences for
U.S. relations with other African nations.
D. The Socialist party leaders in Ganelon believe that their nation‟s interests would best be
served by an alliance with anti-American forces.

Answer Option A

Q.28: It‟s time we stopped searching for new statistics to suggest that we are not
spending enough on education. In fact, education spending increased 30 percent
overall during the last decade.

Which of the following, if true, would most weaken the argument above?

A. Despite increased spending on education, enrollment in our elementary and secondary


schools declined about 4 percent during the last ten years.
B. Our spending on gasoline increased more than 100 percent during the last decade.
C. When adjusted for inflation, our per-pupil expenditure on education this year is less than
it was ten years ago.
D. Eleven other economically developed nations spend more on education than we do.

Answer Option C

Q.29: No nation can long survive unless its people are united by a common tongue. For
proof, we need only consider Canada, which is being torn asunder by conflicts between
French-speaking Quebec and the other provinces, which are dominated by English
speakers.

Which of the following, if true, most effectively challenges the author‟s conclusion?

A. Conflicts over language have led to violent clashes between the Basque-speaking
minority in Spain and the Spanish-speaking majority.
B. Proposals to declare English the official language of the United States have met with
resistance from members of Hispanic and other minority groups.

Website Sources: https://www.edufever.com/ Page 11


Critical Reasoning Set-3
C. Economic and political differences, along with linguistic ones, have contributed to the
provincial conflicts in Canada.
D. Switzerland has survived for nearly a thousand years as a home for speakers of three
different languages.

Answer Option D

Q.30: The U.S. census is not perfect, thousands of Americans probably go uncounted.
However, the basic statistical portrait of the nation painted by the census is accurate.
Certainly some of the poor go uncounted, particularly the homeless; but some of the
rich go uncounted as well, because they are often abroad or traveling between one
residence and another.

Which of the following is an assumption on which the above argument depends?

A. Both the rich and the poor have personal and economic reasons to avoid being counted
by the census.
B. All Americans may reasonably be classified as either poor or rich.
C. The percentage of poor Americans uncounted by the census is close to the percentage
of rich Americans uncounted.
D. The number of homeless Americans is approximately equal to the number of rich
Americans.

Answer Option C

**This Question Paper Set brought to you by Edufever.com**

For More Question Paper, Placement Paper, Tutorial, Study Materials for job
alerts etc.

Visit: www.edufever.com

Follow Us on:

Website Sources: https://www.edufever.com/ Page 12


Critical Reasoning Set-3

Website Sources: https://www.edufever.com/ Page 13


Pr
oudl
yPr
esent
s

C
RI
TI
CA
R
L
E
A
R
S
E
O
N
A
I
S
N
G
O
NIN
G
F
orB
ank
in
g,
SS
C,U
PSC
,St
at
ePS
C,R
ai
lwa
y,
Gen
er
al
Comp
et
i-
ti
ona
ndJ
obr
el
at
edex
ams
.

Partners
Critical Reasoning Set-4
Q.1: In order to combat Carville‟s rampant homeless problem, Mayor Bloomfield
recently proposed a ban on sleeping outdoors in the city‟s many parks. He claims that
such a measure will force the homeless to either leave Carville or to find means other
than sleeping in public parks.

Which of the following, if true, suggests that Mayor Bloomfield‟s plan will be successful?

A. Until the ban, the city‟s many homeless shelters were at less than fifty percent
occupancy.
B. Many homeless tend to congregate underneath Carville‟s numerous overpasses.
C. Adjacent cities have even tougher measures on the homeless sleeping outdoors.
D. The percent of Carville‟s population that has been homeless has been slowly decreasing
in the last five years.

Answer Option A

Q.2: Recent studies have highlighted the harmful effects of additives in food (colors,
preservatives, flavor enhancers etc.). There are no synthetic substances in the foods we
produce at Munchon Foods we use only natural ingredients. Hence you can be sure you
are safeguarding your familys health when you buy our products.

Which of the following, if true, would most weaken the contention of Munchon Foods?

A. Some synthetic substances are not harmful

B. Some natural substances found in foods can be harmful

C. Food without additives is unlikely to taste good

D. Munchon Foods produces only breakfast cereals

Answer Option B

Q.3: Megalimpet is a nationwide owner of office space. They have major office
buildings in the downtowns of several cities in the 48 lower states, and rent this space to
individual companies. Megalimpet office spaces vary from small office to large suites,
and every space has custom-designed wall-to-wall carpeting. The carpet in several
Megalimpet facilities needed replacing. The winning bid for the nationwide carpet
replacement was submitted by Bathyderm Carpet Company (BCC). The bid contract
involves all delivery costs, all installation, and any ongoing maintenance and upkeep
while the carpet is under the three-year warranty. Both BCC executives and
independent consultants they hired felt BCC would be able to perform all these services
for far less than their bid price; these circumstances would allow BCC to reap a
considerable profit.

Website Sources: https://www.edufever.com/ Page 1


Critical Reasoning Set-4
Which of the following, if true, most calls in question the argument that BCC will make a
large profit from this contract with Megalimpet?

A. All the carpets will have to be transported by train from BCC factory in Louisville, KY, to
Megalimpet‟s locations from coast to coast.
B. BCC has already supplied carpets to a number of restaurant chains, and some of those
spaces are as large as Megalimpet‟s largest office spaces.
C. The carpet installation teams will have to cut different sizes of the carpets for the
different size office suites in the Megalimpet buildings.
D. The material in BCC carpets degrades rapidly when it comes into contact with standard
toner, found in most laser printers and photocopiers; the degraded sections are unsightly
and smell bad, so they often need to be replaced.

Answer Option D

Q.4: A fruit known as amla in certain parts of Asia is an excellent source of vitamin C. A
small quantity of the fruit grated and added to salads provides almost all the daily
requirement of this vitamin. However, the fruit is very sour. A new process designed to
remove most of the sour taste will make the fruit acceptable to American tastes. We are
therefore starting to grow this fruit for sale in the United States.

The argument above assumes all of the following except

A. Americans generally wont eat very sour foods


B. The new process does not remove a significant part of the vitamin content
C. That a market exists for a new source of vitamin C
D. The fruit can be used only in salads

Answer Option D

Q.5: Most scientists agree that new lines of interdisciplinary research are the need of
the hour. Even government committees on science have stressed the need for more
interdisciplinary projects. Yet, of ten proposals for new interdisciplinary projects last
year, only one was successfully funded. Some have suggested that this means that as
yet researchers are not coming up with sufficiently persuasive projects, or that their
proposals are not of high enough quality, or even that the reputations of these
researchers is not high enough. However, the real reason probably lies in the way
funding is organized. Funding is still allocated according to the old categories and there
are no funds specifically for research that overlaps different subject areas.

The two parts in bold-face are related to each other in which of the following ways?

A. The first is a finding that the author finds unacceptable; the second is the authors own
position

Website Sources: https://www.edufever.com/ Page 2


Critical Reasoning Set-4
B. The first is a finding that the author attempts to account for; the second is a finding that
contradicts the authors main conclusion.
C. The first is a fact that the author attempts to account for. The second is data that
explicitly supports the authors main conclusion.
D. The first is a position that the author opposes; the second is the authors main position.

Answer Option C

Q.6: Paragraph: A factory was trying out a new process for producing one of its
products, with the goal of reducing production costs. A trial production run using the
new process showed a fifteen percent reduction in costs compared with past
performance using the standard process. The production managers therefore concluded
that the new process did produce a cost savings.

Question: Which of the following, if true, casts most doubt on the production managers'
conclusion?

A. In the cost reduction project that eventually led to the trial of the new process, production
managers had initially been seeking cost reductions of fifty percent.
B. Analysis of the trial of the new process showed that the cost reduction during the trial
was entirely attributable to a reduction in the number of finished products rejected by
quality control.
C. While the trial was being conducted, production costs at the factory for a similar product,
produced without benefit of the new process, also showed a fifteen percent reduction.
D. Although some of the factory's managers have been arguing that the product is outdated
and ought to be redesigned, the use of the new production process does not involve any
changes in the finished product.
E. Since the new process differs from the standard process only in the way in which the
stages of production are organized and ordered, the cost of the materials used in the
product is the same in both processes.

Answer Option C

Q.7: Passengers must exit airplanes swiftly after accidents, since gases released
following accidents are toxic to humans and often explode soon after being released. In
order to prevent passenger deaths from gas inhalation, safety officials recommend that
passengers be provided with smoke hoods that prevent inhalation of the gases.

Which of the following, if true, constitutes the strongest reason not to require
implementation of the safety officials' recommendation?

A. Test evacuations showed that putting on the smoke hoods added considerably to the
overall time it took passengers to leave the cabin.

Website Sources: https://www.edufever.com/ Page 3


Critical Reasoning Set-4
B. Some airlines are unwilling to buy the smoke hoods because they consider them to be
prohibitively expensive.
C. Although the smoke hoods protect passengers from the toxic gases, they can do nothing
to prevent the gases from igniting.
D. Some experienced flyers fail to pay attention to the safety instructions given on every
commercial flight before takeoff.

Answer Option A

Q.8: A minor league baseball franchise experienced a drop in attendance this week
after they suffered three losses by margins of ten runs or more last week. Many
spectators of those games wrote letters to the editors of the local sporting news,
complaining of the poor play of the team in those three losses. Nevertheless, the front
office of this baseball franchise maintains that the team‟s poor play in those three losses
has nothing to do with this week‟s decline in attendance.

Which of the following, if true, most strongly supports the position held by the front office
of the baseball franchise?

A. The spectators who wrote letters to the local sporting news were long-standing fans of
this minor league baseball team.
B. Many minor league baseball franchises attribute a drop in attendance to the quality of
play of the team only after a string of losses.
C. Other minor league teams in that region of the state reported a similar drop in
attendance this week.
D. This was not the first time this team suffered multiple lopsided losses in a single week,
prompting similar letters to the local sporting news.

Answer Option C

Q.9: Anton: I sold my house on an internet site last year and was happy with the price.
I got a speedy sale and the cost of advertising was insignificant. I would advise you to
avoid real estate agents.

Barbie: It is in the interest of the real estate agent to get me the best price for my
property because he gets a commission based on the selling price. Therefore, when
selling my house I will certainly use an agent rather than trying to sell the house by word
of mouth, or by advertising in newspapers or on the internet.

Barbies could strengthen her position by pointing out all of the following except

A. Houses of comparable value often obtain a lower price when sold on the internet
B. Very few houses are sold on the internet at the moment an so a valid comparison is
difficult

Website Sources: https://www.edufever.com/ Page 4


Critical Reasoning Set-4
C. The agents service includes many add-on benefits in terms of legal fees, surveyors
reports and advice that are not available on internet sites
D. Some buyers pay the agent to find them a cheap house

Answer Option D

Q.10: Paragraph: The program to control the entry of illegal drugs into the country was
a failure in 1987. If the program had been successful, the wholesale price of most illegal
drugs would not have dropped substantially in 1987.

Question: The argument in the passage depends on which of the following


assumptions?

A. The supply of illegal drugs dropped substantially in 1987.


B. The price paid for most illegal drugs by the average consumer did not drop substantially
in 1987.
C. Domestic production of illegal drugs increased at a higher rate than did the entry of such
drugs into the country.
D. A drop in demand for most illegal drugs in 1987 was not the sole cause of the drop in
their wholesale price.

Answer: Option D

Q.11: In a few recent cases, some teenagers with advanced programming abilities used
a new programming language, FANTOD, to hack into ETS and change their own SAT
scores. All of the teenagers convicted of this crime were highly skilled in programming
FANTOD. In light of these cases, some colleges have discounted the official SAT
scores of applicants with a knowledge of FANTOD, and have required them to take
special admission tests in supervised conditions on their own campuses.

Which of following conclusions can most properly be drawn from the information above?

A. Most people who learn to program in FANTOD do so to commit some kind of hacking.
B. Colleges should rely on their own admissions tests instead of the SATs
C. The college admission process possibly places some students with knowledge of
FANTOD at a disadvantage.
D. Students who learn FANTOD tend to have much lower SAT scores than do their peers.

Answer Option C

Q.12: Early data on seat-belt use showed that seat-belt wearers were less likely to be
killed in road accidents. Hence, it was initially believed that wearing a seat-belt
increased survival chances in an accident. But what the early analysts had failed to see
was that cautious drivers were more likely to wear the belts and were also less likely to

Website Sources: https://www.edufever.com/ Page 5


Critical Reasoning Set-4
cause big accidents, while reckless drivers were more likely to be involved in big
accidents and were less likely to wear the belts.

Which of the following, if true, could an opponent of the view presented above best cite
as a reason for recommending continued use of seat-belts?

A. Careful drivers who are involved in accidents caused by reckless drivers, would be more
likely to survive if wearing a belt
B. All drivers should be required by law to wear a belt
C. The ratio of big to small road accidents is very small
D. In fatal accidents seat-belt wearers in the front seat are less likely to survive than those
wearing seat belts in the back seat

Answer Option A

Q.13: In the twentieth century, the visual arts have embarked on major experimentation,
from cubism to expressionism. While tastes always vary, there are certainly some
people who find beautiful objects of each of the art movements of the first half of the
twentieth century. In the latter half of the twentieth century, though, most works are so
abstract or shocking that neither the critic nor the general public uses the word
“beautiful” to describe them: indeed, sometimes late twentieth-century artists have, as
one of their expressed goals, the creation of a work that no one could find beautiful.
Whatever these artists are creating may be intellectually engaging at some level, but it
is no longer art.

Which of the following is an assumption that supports drawing the conclusion above
from the reasons given for that conclusion?

A. Art critics generally have a different appraisal of a work of art than does the general
public.
B. The meaning of any work of art is defined entirely by the ideas of the artist who created
it.
C. Beauty is a defining quality of art.
D. All art movements of the latter half of the twentieth century are responses to the
movements of the first half of the century.

Answer Option C

Q.14: Paragraph: Property taxes are typically set at a flat rate per $1,000 of officially
assessed value. Reassessments should be frequent in order to remove distortions that
arise when property values change at differential rates. In practice, however,
reassessments typically occur when they benefit the government—that is, when their
effect is to increase total tax revenue.

Website Sources: https://www.edufever.com/ Page 6


Critical Reasoning Set-4

Question: If the statements above are true, which of the following describes a situation
in which a reassessment should occur but is unlikely to do so?

A. Property values have risen sharply and uniformly.


B. Property values have all risen—some very sharply, some less so.
C. Property values have for the most part risen sharply; yet some have dropped slightly.
D. Property values have for the most part dropped significantly; yet some have risen
slightly.

Answer: Option D

Q.15: French cuisine is highly regarded all over the world. Yet in Paris there are more
American restaurants selling burgers and fries (which many people now class as junk
food) than there are in any other European capital city. Obviously the French are very
fond of junk food, and are not too proud to eat it.

Which of the following, if true, would most weaken the authors contention?

A. There are also a larger number of Lebanese restaurants in Paris than there are in other
European capital cities
B. French Cordon Bleu cuisine is very expensive
C. The number of French tourists eating in New York burger restaurants is very low
D. There are an unusually large number of American tourists in Paris who eat at burger
joints

Answer Option D

Q.16: The National Farm Administration (NFA) has been concerned over the last
decade with the struggles of barley growers.

Plan: In an effort to support these barley growers, two years ago, the NFA began a
program of sending them, each autumn, a free special mix of fertilizer and enzymes
designed to multiply barley yield, to be applied the following spring during first growth.
This mix had been stunningly successful in multiplying the yield of barley in laboratory
conditions.

Results: Most barley growers reported little change in their economic status over this
two year period.

Further information: All barley growers received the shipments, and all used them.
Weather conditions have been fair to optimal for barley growth over the past two years.

Website Sources: https://www.edufever.com/ Page 7


Critical Reasoning Set-4
In light of the further information, which of the following, if true, does most to explain the
result that followed the implementation of the plan?

A. During these two years, most of the barley growers reported using no other fertilizer
besides the special mix sent by the government.
B. The trucks that drove the special mix from the depot in Wisconsin to the individual farms
sometime took as much as 4 or 5 days.
C. Some of the enzymes in the special mix multiply the growth of a bacteria that feeds on
the young barley plants.
D. This program was implemented at a time when more than half of barley growers
nationwide were reported barely breaking even in their yearly expenses.

Answer Option C

Q.17: Paragraph: To persuade consumers to buy its personal computers for home use,
SuperComp has enlisted computer dealers in shopping centers to carry its product and
launched a major advertising campaign that has already increased public awareness of
the SuperComp brand. Despite the fact that these dealers achieved dramatically
increased sales of computers last month, however, analysts doubt that SuperComp‟s
products accounted for much of that increase.

Question: Which of the following, if true, best supports the claim that the analysts‟
doubt is well founded?

A. In market surveys, few respondents who had been exposed to SuperComp‟s advertising
campaign said they thought there was no point in owning a home computer.
B. People who own a home computer often buy a second such computer, but only rarely do
people buy a third computer.
C. SuperComp‟s dealers also sell other brands of computers that are very similar to
SuperComp‟s but less expensive and that afford the dealers a significantly higher
markup.
D. The dealers who were chosen to sell SuperComp‟s computers were selected in part
because their stores are located in shopping centers that attract relatively wealthy
shoppers.

Answer: Option C

Q.18: Linda: In the 1800s, it was found that one in every six women who gave birth in
hospitals died of a fever they had contracted after delivering the child and that the
mortality was not as high if they gave birth at home with the help of a mid wife. It was
found that the doctors had a poorer sense of hygiene and that their dirty hands and
instruments were leading to pathogens entering a woman's bloodstream. Thankfully,
hygienic conditions today are much better and women are safer.

Website Sources: https://www.edufever.com/ Page 8


Critical Reasoning Set-4
Fiona: But doctors today are so overworked that a number of doctors, while aware of
the need for better hygiene, barely find the time to wash their hands. The likelihood of
infections caused by doctors is probably not any better.

Q. Which of the following can be used by Fiona to further establish that Linda need not
be correct in her reasoning?

A. A nationwide survey found that doctors, especially experienced ones, are more prone to
the belief that they could not possibly be carrying pathogens.
B. It is reported that around 50,000 people die every year in our country from hospital
acquired infections.
C. Compulsory hand cleansing is strictly adhered to by all doctors and nurses before every
major surgical procedure.
D. During a study, doctors handling newborn babies self reported a hand cleansing rate of
90℅.

Answer Option A

Q.19: Alan: In the last 15 years, most of the criminals who were convicted of theft or
murder were from the lower income classes and had not completed high school.
Therefore, the government has to spend more money on reducing poverty and increase
funding to education. Because terrorism is the most severe of all crimes, such
measures would bring down overall crime rate and reduce threat from terrorism.

Dylan: A study that was conducted in a country known to produce a number of terrorists
showed that on average the terrorists were better educated than the overall population
and that they did not necessarily come from lower income classes. This is probably
because crimes such as theft are committed for personal gain while terrorism is for
political or religious gain.

Which of the following best describes Dylan's response to Alan?

A. Dylan changes the direction of the argument entirely by discussing the scenario in a
different country
B. Dylan partially agrees with Alan's reasoning but refutes his recommendation to the
government
C. Dylan converts a causal argument made by Alan into a generalization applicable
universally
D. Dylan challenges Alan's reasoning by explaining why two situations that Alan perceives
as similar are not

Answer Option D

Website Sources: https://www.edufever.com/ Page 9


Critical Reasoning Set-4
Q.20: The Americans with disabilities act (ADA) was designed to ensure that there is no
discrimination against and unfair termination of differently-abled workers in the
workplace. However, after the act was introduced, there has been a marked increase in
unemployment among the differently-abled.

Which of the following best explains this seeming discrepancy?

A. A number of differently-abled people chose not to work


B. Not willing to deal with the issues of workplace discrimination of the differently-
abled, several companies recruited fewer of them in the first place
C. Knowing that the act was about to be enforced, companies terminated some of
the differently-abled while they had a chance
D. There was no act introduced that would guarantee a job for the differently-abled

Answer Option B

Q.21: Studies have established that children who watched 2 more hours of TV on an
average daily basis during the first 15 years of their life were 50℅ more likely to be
arrested for property crimes in the country. Researchers believe that these studies
clearly establish that violence in movies and TV contribute to aggressive behavior in
real life. On the other hand, there is no clear evidence that the programs that the kids
watched on TV were violent in the first place. Even if we were to accept that TV
watching contributed to the increased crime rate, it need not have been because of the
nature of the programs. Perhaps, children who watched programs such as Adams and
Samson, a funny sit-com about two blundering cops began perceiving all cops as
incompetent.

What is the role of the sentences in boldface?

A. The first establishes a theory that the author later clearly refutes while the second
presents the author's conclusion
B. The first is an interpretation made by researchers that the author contends against and
the second provides an alternative explanation for statistical data
C. The first is an opinion expressed by someone other than the author and which the author
is completely in disagreement with while the second provides the reason for the author's
doubt
D. The first is a fact supporting the researcher's conclusion while the second is the author's
conclusion.

Answer Option B

Q.22: In an effort to curb drug abuse, the government has imposed strict laws to
prosecute the dealers. However, such an initiative is unlikely to be effective.

Website Sources: https://www.edufever.com/ Page 10


Critical Reasoning Set-4
Prosecuting dealers will lead to a shortage of drugs. At the same time, because no
efforts are being taken to curb demand, drugs will be sold at a premium, attracting more
people to the very remunerative job of drug dealing. Therefore, to effectively reduce
drug abuse, the government will have to prosecute the drug users and not dealers.

Which of the following is the most relevant information in evaluating the credibility of the
argument?

A. Whether efforts have been successfully taken in any other country to regulate drug
users.
B. Whether the payoff from selling drugs outweighs the severity of the punishment
C. Whether drugs will continue to be sold at a premium when there are dealers in the
market again
D. Whether the majority of the users will be willing to pay a premium to continue to use the
drugs

Answer Option D

Q.23: During mediaeval times, the administrative system was organized such that jobs
were traditionally held within the same family. The eldest son of the village's blacksmith
will take up his father‟s business and become the next blacksmith. The other sons
would join the army or serve the king in some fashion while the daughters did what their
mother did. Although the world has undergone innumerable changes, the dynastic
system has not undergone any change whatsoever. Children who have fathers who
played major league baseball are 800 times more likely than other kids to become major
league players themselves.

Which of the following best refutes the author's reasoning?

A. In countries with a royal family, the eldest son of the king is destined to be the next king.
B. The blacksmith's eldest son in a mediaeval village could not choose any other profession
even if he wanted to.
C. A major baseball player will have better knowledge and skills and will be able to guide
his son better.
D. 60℅ of the country's doctors have at least one parent who is a doctor.

Answer Option B

Q.24: During the last 50 years in England, the national football team has had at least
60℅ of its players born during the months January to March. Similarly, in Germany, at
least 50℅ of the team has been born during the first three months of the year. In fact,
this statistic holds true for most European countries. This shows that in most European

Website Sources: https://www.edufever.com/ Page 11


Critical Reasoning Set-4
countries, parents with kids born early in the year are more likely to encourage a football
career.

Which of the following best explains why the conclusion need not be the best
explanation for the statistic?

A. Of the remaining members, 60℅ or more tend to be born between the months of April
and June.
B. Parents of kids born in the second half of the year have displayed a measurable intent to
promote scholarly careers
C. The age cut-off for kids to try out for a team in most European countries is calculated as
on December 31 of any year and older the kid during trials, greater the chances of
getting the right opportunities.
D. The school year usually begins in April in most European countries and many parents
are known to encourage students to work on their academics and not sports during the
school year.

Answer Option C

Q.25: Terrorist attacks invariably lead to tremendous losses in life, property, and morale
of a country. The effects of a terrorist attack are not just immediate and can have long-
lasting, trickle-down effects as well. The fear, for example, takes a long time to die
down. However, some of these repercussions can be beneficial to the country. Take for
instance, the recent terrorist attack on our capital city. In the weeks following the attack,
the crime rate in the city came down significantly from what it was just before the attack.
This must primarily be due to the increased presence of police resources that were
moved to the area and is thus an indirect effect of the attack.

Which of the following options gives one more option as to why the crime rate
decreased because of the terrorist attacks?

A. The capital city is under increased monitoring leading to quick detection of crimes -
many times while the crime is still underway.
B. A number of people are frightened because of the terrorist attacks and have fled the
capital city.
C. There was a recorded decrease in crime rate right after the terrorist attack in almost all
cities of the country.
D. Intel reports show that the terrorists who pulled off the attack had been committing
various other smaller crimes regularly to distract law enforcement from their true
purposes.

Answer Option D

Website Sources: https://www.edufever.com/ Page 12


Critical Reasoning Set-4
Q.26: In 2009, there was a sharp increase in the number of people who were reported
to have died in the country during the first week of the year, compared with the first
week of the previous ten years. There seems to be no reason to explain this
disproportionately high number of deaths especially because the reports were
widespread in the country and there was no epidemic spreading through the country at
that point in time. Moreover, most of the deaths were not among the young and could
not even be attributed to binge drinking in celebration of the new year.

Which of the following can then explain the discrepancy?

A. The government announced in 2008 that estate taxes, payable by the heirs on
someone's death, would be abolished effective from the new year.
B. Astrologists predicted that 2009 was a good year to die in, for salvation of the soul.
C. The statistic is an anomaly that has no logical explanation.
D. The price of some cancer drugs and chemotherapy increased by 10% in January 2009,
making treatment more expensive than before.

Answer Option A

Q.27: Industrial and automobile pollution have long been thought to contribute to global
warming. However, researchers have identified that the carbon dioxide and carbon
monoxide released into the atmosphere are not as potent as the methane emitted by
cud-chewing animals such as cows when they fart or belch. Therefore, if you drive a
hybrid electric car to the grocery, any favor that you would do to the environment would
be offset if you end up buying beef.

Which of the following best describes the author's reasoning?

A. The author establishes a point by drawing an analogy


B. The author disproves a popular notion by providing evidence that is contrary to it
C. The author presents a new theory in response to an existing well-established theory
D. The author uses an illustration to support a new school of thought that is contrary to a
popular school of thought

Answer Option D

Q.28: There are several scientific studies and research findings that are constantly
discussed and publicized in the news media. Some of these are contrary to other
research studies that are published. This conflict in information makes people believe
that either science is inaccurate or that they can „choose‟ which scientific result they
want to believe in. However, neither of these beliefs is accurate. Science is not
inaccurate or subjective. Most of the time, scientific studies show inconsistent results
either because of inaccuracies in methodology adopted or because of

Website Sources: https://www.edufever.com/ Page 13


Critical Reasoning Set-4
misrepresentation of actual results by the news media. For example, a recent study
done on just 15 women showed that eating chocolate was not necessarily harmful
during pregnancy and the media reported that chocolate was actually beneficial to the
foetus.

Which of the following best further corroborate the author‟s argument?

A. A study result that shows that the effect of automobile pollution on global warming is
marginal.
B. An unverified study gaining popularity because it seemingly showed that red wine
reduces the risk of cancer on a very small control group.
C. A new study that further collaborates an earlier finding that the consumption of some
types of berries lower the risk of heart failure.
D. The news media highlighting the fact that a study had clearly proven that effects of
certain drugs on rats need not be the same as on humans.

Answer Option B

Q.29: John: A study has identified that many soldiers received less-than-honorable
discharges from the army due to charges of misconduct that can actually be attributed
to conditions such as PTSD and traumatic brain injury. This is a military practice that is
unfair and must be avoided. The army must take responsibility for the same and stop
discharging their soldiers dishonorably.

Christy: While it is true that it is unfair to the soldiers to be discharged so from the
army, it has also been found that there are no sufficient measures or systems that have
been created to identify whether the misconduct was due to trauma or due to other
reasons. Moreover, to enforce discipline in the ranks, it is important for the army to
follow a uniform rule for all the soldiers.

Which of the following statements can Christy further add to her argument?

A. If there is no proper system, then the army should have developed a proper system by
now.
B. Complete elimination of dishonorable discharges would make it more difficult for the
army to monitor and evaluate the soldiers.
C. It is not the army‟s fault that there has been no system developed to help those with
PTSD
D. Studies have also found that showing consideration for some soldiers has led to others
taking advantage, indirectly promoting misconduct.

Answer Option D

Website Sources: https://www.edufever.com/ Page 14


Critical Reasoning Set-4
Q.30: Jess: To be a woman in this century is far better than to be a woman in centuries
past. Life expectancy for women has - for the first time - surpassed that for men and
while only around 20℅ of college students in the late 1800s were women, today, almost
60℅ of college students are women.
Sam: However, women educated in the top-ranked colleges in the country earn only
about 70℅ of what their male counterparts earn. Also, studies show that even today, in
a number of industries, women are refused top management positions simply because
they are women.

Which of the following is Jess most likely to say in response to Sam?

A. Women today are healthier and live longer than women in the 1800s, who primarily died
because of child birth.
B. There are more women at the entry-level jobs of most industries than there are men.
C. Women today are better able to juggle professional and personal lives and a number of
them are proving to be successful single mothers.
D. The proportion of women in the judicial and legislative branches of the country is at its
highest and the country elected its first woman president recently.

Answer Option D

**This Question Paper Set brought to you by Edufever.com**

For More Question Paper, Placement Paper, Tutorial, Study Materials for job
alerts etc.

Visit: www.edufever.com

Follow Us on:

Website Sources: https://www.edufever.com/ Page 15


Critical Reasoning Set-4

Website Sources: https://www.edufever.com/ Page 16


Pr
oudl
yPr
esent
s

C
RI
TI
CA
R
L
E
A
R
S
E
O
N
A
I
S
N
G
O
NIN
G
F
orB
ank
in
g,
SS
C,U
PSC
,St
at
ePS
C,R
ai
lwa
y,
Gen
er
al
Comp
et
i-
ti
ona
ndJ
obr
el
at
edex
ams
.

Partners
Critical Reasoning SET 5

1. The number of people diagnosed with dengue fever (which is contracted from the
bite of an infected mosquito) in North India this year is twice the number diagnosed
last year. The authorities have concluded that measures to control the mosquito
population have failed in this region.

All of the following, if true, would cast doubt on the authorities conclusion except

A. more cases are now reported because of increases in administrative efficiency


B. a very high proportion of the cases were in people who had recently returned from
neighboring countries
C. an effective diagnostic test was introduced about nine months ago
D. the disease is prevalent only in some industrialized areas which have shown a
dramatic increase in population due to migration
E. the incidence (number of cases per thousand) of malaria, also contracted from
mosquito bites, has increased

Answer: Option E

2. Photography is no longer an art form. Nowadays everyone has access to digital


cameras that only need to be pointed at the subject in order to generate a perfect
image.

The writer of the argument apparently assumes that

A. The selection of the subject is not an important artistic factor in photography


B. Digital cameras will continue to improve in quality
C. Digital cameras can never go wrong
D. Photography with all other types of camera is an art form
E. Art is not perfect

Answer: Option A

3. Red is a color which has powerful effects on human beings as well as animals. A
group of psychologists carried out an experiment which confirms the subconscious
effects of this color on human behavior. They provided selected sports teams at
school and college level with either red or blue shorts and recorded the outcome of
the games. The teams wearing red won in a disproportionate number of matches. The
psychologists suggested that either the teams wearing red subconsciously felt
themselves more powerful, or that the non-red teams were subconsciously
intimidated by the red color.

Which of the following, if true, would most weaken the psychologists suggestion?

A. Each team wore red in some matches and blue in others.


B. The color blue has the subconscious effect of making human beings less
competitive.
C. The effect was only observed if all the team members wore white shirts.
D. Red signifies danger in some cultures whereas it signifies happiness in others.
E. In a subsequent study, teams with all-red outfits were more likely to report that they
thought they would win no matter what color the opponents wore.

Website Sources: https://www.edufever.com/ Page 1


Critical Reasoning SET 5

Answer: Option B

4. The enormous distances between stars are not spaces entirely devoid of matter.
The interstellar spaces are filled with dust: very low density matter. This miniscule
amount of matter, spread over almost infinite distances, acts like a curtain obscuring
the stars that lie behind. If it were not for this material we would see no dark patches
in the sky at night: the sky would be entirely covered with stars.

The two parts in boldface play what roles in the argument above?

A. The first is a suggestion that the author wishes to dispute. The second is hypothesis
that the author wishes to explain.
B. The first is the main point the author wishes to make. The second is a hypothetical
result of accepting that point.
C. The first is a fact that the author thinks is important in explaining a certain
phenomenon. The second is a result that the author would expect if that fact were not
true.
D. The first is a speculation that the author wishes to justify. The second is a
consequence that would result if that speculation is not true.
E. The first is an established fact that the author wishes to explain. The second is a
consequence of accepting this fact.

Answer: Option A

5. The statement that premises are intended to support is called.

A. A related premise
B. An argument
C. A description
D. The conclusion

Answer: Option D

6. Critical thinking concerns…

A. Determining the cause of our beliefs


B. Pinpointing the psychological basis of our beliefs
C. Determining the quality of our beliefs
D. Assessing the practical impact of our beliefs

Answer: Option C

7. According to the text, critical thinking complements…

A. Our prejudices
B. Our emotions
C. Peer pressure
D. Our unconscious desires

Answer: Option B

Website Sources: https://www.edufever.com/ Page 2


Critical Reasoning SET 5

8. A belief is worth accepting if…

A. We have good reasons to accept it


B. It is consistent with our needs
C. It has not been proven wrong
D. It is accepted by our peers

Answer: Option D

9. Statements backed by good reasons are…

A. Worthy of strong acceptance


B. To be believed with certainty
C. Never false
D. Beyond all possible doubt

Answer: Option A

10. A word that is not a premise indicator word is.

A. Therefore
B. As
C. Since
D. For

Answer: Option A

11. The word critical in critical thinking refers to…

A. A fault-finding attitude
B. Attempts to win an argument
C. Using careful judgment or judicious evaluation
D. A lack of respect for other people

Answer: Option C

12. The statements (reasons) given in support of another statement are called…

A. An argument
B. The conclusion
C. The premises
D. The complement

Answer: Option C

13. A statement is…

A. A question or exclamation
B. An affirmation of prior beliefs
C. An assertion that something is or is not the case
D. An assertion that is neither true nor false

Answer: Option C

Website Sources: https://www.edufever.com/ Page 3


Critical Reasoning SET 5

14. The process of reasoning from a premise or premises to a conclusion based on


those premises is known as.

A. Extended reasoning
B. Subordinate premise
C. Dialectic
D. Inference

Answer: Option D

15. Statements given in support of another statement are called.

A. Conclusions
B. Premises
C. Arguments
D. Summaries

Answer: Option B

16. Words that frequently accompany arguments and signal that a premise or
conclusion is present is called.

A. Inference words
B. Premise indicators
C. Indicator words
D. Equivalent words

Answer: Option D

17. It is often thought that our own modern age is unique in having a large number of
people who live into old age. It has frequently been assumed that plagues, wars, and
harsh working conditions killed off most people in previous ages before they could
reach old age. However, recent research shows that in 17th century Europe, for
example, people over sixty comprised 10 percent of the population. The studies also
revealed that although infant mortality remained high until the 20th century in Europe,
people who survived to adulthood could expect to live to be old.

The portions in boldface play which of the following roles in the argument above?

A. The first is a conclusion that the author supports. The second is data that contradicts
that conclusion.
B. The first is a finding that the author contests. The second is a finding that the author
accepts.
C. The first is an assumption that the author thinks is invalid. The second is data that
validates that assumption.
D. The first is a position that the author opposes. The second is a finding that supports
the author s position.
E. The first is a position that the author opposes. The second is an assumption which, if
valid, negates the author s view.

Answer: Option D

Website Sources: https://www.edufever.com/ Page 4


Critical Reasoning SET 5

18. A marriage counselor noted that couples who have occasional violent arguments
are less likely to divorce within the next six months than those who have frequent but
less violent arguments. He concluded that frequent arguing is a major factor in the
causation of severe marital disharmony.

The counselor s conclusion is most weakened by which of the following


observations?

A. Couples who have already come to the point of divorce argue continuously over
small matters.
B. People who have recently divorced are more likely to argue violently when they meet.
C. Many people in happy marriages have occasional violent arguments.
D. Recently divorced people rarely cite frequent arguments as a cause of marital
disharmony
E. A significant f

Answer: Option A

19. Drinking alcohol is bad for health. Yet a lot of people drink. The above statement
resembles to which of the following?

A. Drinking alcohol makes a person happy. Thus a lot of people drink it.
B. Marriage is full of tension. Yet a lot of people get married.
C. Smoking questions social norms. Thus, men hardly smoke in our country.
D. Rakibul is tall. Yet he never played basketball his whole life.
E. Rezwan thinks he is intelligent. Yet, he never does well in the tests.

Answer: Option B

20. A classic red Ferrari is beautiful, even if there is no one to appreciate it. Logically,
this statement would contradict with which of the following?

A. Beauty is relative.
B. You see what the mind wants you to see.
C. Beauty exists only in the eye of the beholder.
D. There is no accounting for choice.
E. The contemplation of beauty is the greatest pleasure available to us.

Answer: Option B

Website Sources: https://www.edufever.com/ Page 5


Critical Reasoning SET 5

21. Sourodip – “Only regular council members sit on the higher level committee
meetings”

Zihan – “That’s not true; Shams is a regular council member and he has not been a
part of the higher level committee meetings”

Zihan’s response implies that he incorrectly interpreted Sourodip’s statement to mean


that

A. Everyone who’s a part of the higher level committee meetings is a regular council
member.
B. Shams takes part in the higher level committee meetings.
C. No regular council member takes part in the higher level committee meetings.
D. All regular council members take part in the higher level committee meetings.

Answer: Option D

22. Rakibul must be a football player. I saw him wearing a football jersey the other
day.

The conclusion above is valid only if it is true that _______________.

A. Football players often wear football jerseys.


B. All footballers wear football jerseys.
C. Footballers never wear any kind of jerseys other than football jerseys.
D. Only footballers wear football jerseys.

Answer: Option D

23. Only the clever become rich. Which of the following contradicts with the above
statement?

A. Both “clever” and “rich” are relative terms.


B. Tabasser was clever, yet he was poor his whole life.
C. Mehedi is not clever, yet he amassed a large fortune by the time he was 30.
D. Some clever people do not want to get rich

Answer: Option C

24. Nowadays it is very important to teach students to use smartphones effectively.


Therefore students should be taught Python programming in school.

Which of the following if true most weakens the argument above?

A. Only people who use smartphones effectively are skilled at Python


programming.
B. Only people skilled at Python programming use smartphones effectively.
C. Some people who use smartphones effectively cannot write Python programs.
D. Some schools teach Python programming more effectively.

Answer: Option C

Website Sources: https://www.edufever.com/ Page 6


Critical Reasoning SET 5

25. Rasheeq thinks that he is intelligent. But that is not true since he has not managed
to get into IBA. The above argument is based on which of the following assumptions?

A. Most students that get into IBA are intelligent.


B. Being intelligent is not a prerequisite for getting into IBA.
C. Not all intelligent students get into IBA.
D. Students who don’t manage to get into IBA are not intelligent.

Answer: Option D

26. All good footballers want to win, and all footballers who want to win exercise
regularly; therefore, all footballers who do not exercise regularly are bad footballers.

If the assumptions of the arguments above are true, then which of the following
arguments must be true?

A. No bad footballer wants to win.


B. No footballer who does not exercise regularly is a good footballer.
C. Every footballer who exercises regularly is a good footballer.
D. All footballers who want to win are good footballers.

Answer: Option B

27. Which of the following, if true, would weaken the conclusion drawn in the above
argument?

A. Those who are already using the above new products report greater difficulty in
transition from new products to regular ones than from the regular ones to the new
ones.
B. The cost of manufacturing these new products is not more than the cost of
manufacturing the regular ones and the new products last longer than the regular
ones.
C. The number of offices using the new products is increasing month by month.
D. These products need to be purchased in huge lots and need to be stored in special
conditions. The cost of procurement and strong is quite high.

Answer: Option D

28. The number of instances where judges have made unwanted remarks in open
courts against other constitutional/ statutory bodies or persons who were not before
them is increasing. There is a need to bring such behaviour of judges within the
purview of judicial standards.

Which of the following is an assumption implicit in the above passage?

A. There is a set of judicial standards which the judges are expected to follow.
B. Till now no one has set any standards for judges.
C. It is not desirable for the judges to pass such comments on other constitutional/
statutory bodies in their absence.
D. None of These

Answer: Option C

Website Sources: https://www.edufever.com/ Page 7


Critical Reasoning SET 5

29. It may prudent to bear with a small harm for a greater benefit. Problem arises
when the harm has the power to endanger future generations. Research has
consistently shown that frequent exposure X-rays can cause cells in human body to
be destroyed or mutated, apart from this it can cause damage to DNA cell structure, a
bane that could be passed down for generations. Doctors, unmindful of such
consequences, subject the patients to exposure to X-rays, even in the cases in which
it can be avoided.

Which of the following can be concluded/ inferred from the above passage?

A. Doctors think of other consequences of the treatment they provide to the patients.
B. All the hospitals have X-ray machines.
C. There are no government directives regarding the manufacture of X-ray machines.
D. None of these

Answer: Option D

30. Read the above passage, answer the below question.

Which of the statements A, B, C, D and E mentioned above is a conclusion that can be


drawn from the above statement?

A. A
B. B
C. C
D. D
E. None of these

Answer: Option E

**This Question Paper Set brought to you by Edufever.com**

For More Question Paper, Placement Paper, Tutorial, Study Materials for job
alerts etc.

Visit: www.edufever.com

Follow Us on:

Website Sources: https://www.edufever.com/ Page 8

You might also like